Thanks to visit codestin.com
Credit goes to www.scribd.com

0% found this document useful (0 votes)
304 views92 pages

Rotational Problem and Solutions

The book 'Problems and Solutions in Rotational Mechanics' by Pradeep Kumar Sharma provides a comprehensive overview of rotational mechanics, including theories, problems, and solutions aimed at students preparing for competitive examinations like IIT-JEE and Physics Olympiads. It covers various topics such as kinematics of rotation, torque, angular momentum, and energy of rigid bodies, with detailed explanations and systematic problem sets. The author emphasizes the need for concise and well-structured resources to aid both students and teachers in mastering physics concepts.

Uploaded by

sudiptomondal
Copyright
© © All Rights Reserved
We take content rights seriously. If you suspect this is your content, claim it here.
Available Formats
Download as PDF, TXT or read online on Scribd
0% found this document useful (0 votes)
304 views92 pages

Rotational Problem and Solutions

The book 'Problems and Solutions in Rotational Mechanics' by Pradeep Kumar Sharma provides a comprehensive overview of rotational mechanics, including theories, problems, and solutions aimed at students preparing for competitive examinations like IIT-JEE and Physics Olympiads. It covers various topics such as kinematics of rotation, torque, angular momentum, and energy of rigid bodies, with detailed explanations and systematic problem sets. The author emphasizes the need for concise and well-structured resources to aid both students and teachers in mastering physics concepts.

Uploaded by

sudiptomondal
Copyright
© © All Rights Reserved
We take content rights seriously. If you suspect this is your content, claim it here.
Available Formats
Download as PDF, TXT or read online on Scribd
You are on page 1/ 92

Problems and Solutions in

Rotational Mechanics

Online at: https://doi.org/10.1088/978-0-7503-6472-0


Problems and Solutions in
Rotational Mechanics
Pradeep Kumar Sharma MSc, PhD, CEng(I) MIET, MInstP, SMIEEE
Consultant physicist and researcher, (India)

IOP Publishing, Bristol, UK


ª IOP Publishing Ltd 2024. All rights, including for text and data mining (TDM), artificial
intelligence (AI) training, and similar technologies, are reserved.

This book is available under the terms of the IOP-Standard Books License

No part of this publication may be reproduced, stored in a retrieval system, subjected to any
form of TDM or used for the training of any AI systems or similar technologies, or transmitted
in any form or by any means, electronic, mechanical, photocopying, recording or otherwise,
without the prior permission of the publisher, or as expressly permitted by law or under terms
agreed with the appropriate rights organization. Certain types of copying may be permitted
in accordance with the terms of licences issued by the Copyright Licensing Agency, the Copyright
Clearance Centre and other reproduction rights organizations.

Permission to make use of IOP Publishing content other than as set out above may be sought
at [email protected].

Pradeep Kumar Sharma has asserted his right to be identified as the author of this work in
accordance with sections 77 and 78 of the Copyright, Designs and Patents Act 1988.

ISBN 978-0-7503-6472-0 (ebook)


ISBN 978-0-7503-6468-3 (print)
ISBN 978-0-7503-6469-0 (myPrint)
ISBN 978-0-7503-6471-3 (mobi)

DOI 10.1088/978-0-7503-6472-0

Version: 20241101

IOP ebooks

British Library Cataloguing-in-Publication Data: A catalogue record for this book is available
from the British Library.

Published by IOP Publishing, wholly owned by The Institute of Physics, London

IOP Publishing, No.2 The Distillery, Glassfields, Avon Street, Bristol, BS2 0GR, UK

US Office: IOP Publishing, Inc., 190 North Independence Mall West, Suite 601, Philadelphia,
PA 19106, USA
This book is dedicated to
All the teachers, physicists and lovers of physics
And
My Revered Guru of Metaphysics (Vedic Philosophy)
Professor Shankha Purohit, MA(Odia), MA(English), MA(Sanskrit).
Contents

Foreword xi
Preface xii
Acknowledgements xvi
Author biography xx

1 Kinematics of rotation 1-1


1.1 Definition of a rigid body 1-1
1.2 Translation of a rigid body 1-2
1.3 Angular motion or rotation of a rigid body 1-3
1.4 Angular velocity of a rigid body 1-4
1.5 Calculation of the angular velocity 1-4
1.5.1 Method 1: ground frame method 1-4
1.5.2 Method 2: relative motion method 1-5
1.6 Angular acceleration and its calculation 1-6
1.6.1 The ground frame method 1-6
1.6.2 Relative motion method 1-7
1.6.3 Gyroscopic motion 1-9
1.7 Fixed axis rotation 1-9
1.7.1 Centroidal 1-10
1.7.2 Non-centroidal 1-10
1.8 Combined motion 1-11
1.9 Finding the velocity and acceleration of a point of a rigid body 1-12
under combined motion
1.10 The kinematics of rolling 1-16
1.11 General kinematical misconceptions 1-21
1.12 Kinematical equations for uniform angular acceleration 1-22
1.13 The instantaneous axis of rotation (IAR) 1-24
1.13.1 Calculation of the position of the IAR 1-24
1.14 Rotation about a point and gyroscopic motion 1-31
Problems 1-33

2 Torque and angular momentum for a point mass 2-1


2.1 Torque 2-1
2.1.1 Definition 2-1

vii
Problems and Solutions in Rotational Mechanics

2.1.2 Factors governing the torque 2-2


2.1.3 The mathematical expression of torque 2-2
2.1.4 Net torque when many forces act at a point 2-4
2.1.5 Torque about an axis 2-6
2.2 Angular momentum 2-9
2.2.1 Nomenclature 2-9

2.2.2 The magnitude of L 2-10
2.2.3 The direction of angular momentum 2-11
2.2.4 The concept of turning 2-12
2.2.5 The relation between angular momentum L and angular 2-13
velocity w
2.2.6 Angular momentum about an axis 2-15
2.3 Relation between torque and angular momentum 2-17
2.4 Conservation of the angular momentum of a particle 2-17
Problems 2-18

3 The statics and dynamics of rotation 3-1


3.1 Torque acting on a group/system of particles 3-1
3.2 Torque about the center of mass 3-3
3.3 The angular momentum of a system of particles 3-6
3.4 Angular momentum of a two-particle system about the center of mass 3-8
3.5 The relation between the relative and absolute values of angular 3-10
momentum relative to two coinciding reference frames
3.6 The relation between torque and angular momentum 3-12
3.7 Newton’s laws for a system of particles 3-14
3.8 Conservation of the angular momentum of a system of particles 3-16
3.9 The angular momentum of a rigid body about the center of mass 3-19
3.10 The moment of inertia and its calculation 3-20
3.10.1 Discrete particle system 3-20
3.10.2 The moment of inertia due to continuous mass distribution 3-23
3.10.3 Parallel axis theorem 3-30
3.10.4 Perpendicular axis theorem 3-33
3.11 Newton’s laws of motion of rigid bodies 3-35
3.11.1 Law of rotation 3-35
3.11.2 Law of translation 3-36
3.12 Equilibrium of rigid bodies 3-39
3.12.1 Translation of rigid bodies 3-39
3.12.2 Rotational equilibrium 3-42

viii
Problems and Solutions in Rotational Mechanics

3.13 Fixed axis rotation 3-43


3.13.1 Newton’s law of rotation 3-43
3.13.2 Newton’s law of translation 3-45
3.13.3 Kinematics 3-46
3.14 Dynamics of the combined motion of rigid bodies 3-47
3.14.1 Planner motion: Newton’s law of rotation 3-47
(torque equation)
3.14.2 Newton’s law of translation (force equation) 3-47
3.15 Gyroscopic motion 3-48
Problems 3-51

4 The energy of rigid bodies 4-1


4.1 The gravitational potential energy of rigid bodies 4-1
4.2 Kinetic energy of rigid of a body 4-2
4.2.1 Discrete mass distribution 4-2
4.2.2 Continuous mass distribution 4-4
4.2.3 Rotational and translational kinetic energy 4-6
4.3 The kinetic energy of rolling bodies on a fixed surface 4-9
4.4 Work done by a force on a rigid body 4-12
4.5 The work–energy theorem for rigid bodies 4-14
4.5.1 Net work done on a rigid body 4-14
4.5.2 Work–kinetic energy theorem 4-18
4.6 Conservation of energy of rigid bodies 4-20
4.7 Power delivered by a force acting on a rigid body 4-21
Problems 4-23

5 Impulse and momentum of rigid bodies 5-1


5.1 The linear momentum of a rigid body 5-1
5.2 Angular momentum of a rigid body 5-1
5.2.1 Spin angular momentum 5-1
5.2.2 Orbital angular momentum 5-2
5.2.3 Total angular momentum 5-2
5.3 The impulse–momentum equation 5-3
5.3.1 The linear impulse–momentum equation 5-3
5.3.2 The angular impulse–momentum equation 5-4
5.4 Conservation of angular momentum 5-7

ix
Problems and Solutions in Rotational Mechanics

5.5 The collision of rigid bodies 5-10


5.5.1 The collision of a particle with a rigid body 5-10
5.5.2 Collison of a rigid body with another rigid body 5-15
Problems 5-15

x
Foreword

It is my pleasure to write a foreword for this book authored by Dr Pradeep Kumar


Sharma. The author of this book knew me while he was a lecturer of Physics at
Brilliant Tutorials (BT), Chennai, and I was a professor of Physics at the Indian
Institute of Technology (IIT), Madras. I also know the author for his series of five
books—GRB Understanding Physics. The author has gained vast experience and
expertise in training the best students for various competitive examinations such as
IIT-JEE, Physics Olympiads, etc. In 2020, the author personally visited my house in
Chennai and requested me to edit this book. Although I could not edit, I spent some
time in reviewing the matter and gave some suggestions to improve the overall
quality and content of this book.
This book is well-written and well-balanced with theories and problems. All
concepts are covered in-depth. The explanations have been presented in detail in a
simple and lucid style. The most appropriate examples are given by the author based
on his vast experience of teaching physics in reputed institutes across the country
such as BT, FIIT-JEE, etc. This book has an impressive layout and excellent quality
of production. In each chapter, the author has included systematic theories with the
best examples and scholarly set solved problems. This will assist both teachers and
students in building and strengthening their concepts of physics.
This book will be immensely useful to all college students preparing for entrance
examinations such as IIT-JEE and Physics Olympiads, etc. Furthermore, this book
could be useful for the physics GRE and PhD qualifying examinations for top
universities across the world. Teachers can also use this book to enhance their
subject knowledge so as to impart a better physics education. Each topic is dealt with
scrupulously so as to enhance the student’s understanding of the subject to a great
extent. I thank the author for this splendid work as the result of his hard work and
determination. I wish him all the best for his forthcoming books.

Dr Jagabandhu Majhi
Professor of Physics (Retired)
Indian Institute of Technology, Madras

xi
Preface

Overview of the series


There are a lot of books on the market; each has its own merits and limitations. It is
not possible to get everything into a single book. However, a potential student
expects everything in a book, such as a variety of adequate problems with lucid
theories and solved examples. Most books are bulky with too many unnecessary
theories and problems. Currently, the market is flooded with numerous textbooks
with unscholarly repeated questions, many containing only exercises without any
depth or sequence; additionally, thousands of common repeated problems are freely
available on the Internet, and many institutions have their own study materials,
mostly compiled by unscrupulous cut-and-paste methods over many years. Students
are confused by the scattered variety of sources for problems, as well as their
provenance and the validity of solutions. Thus, not only do the helpless students but
also their teachers need some concise and precise books containing exercises where
the primary concepts are covered systematically. This series of books is a sincere
effort to minimize all the above limitations of existing books and resources and
maximize the potential of our books in terms of content, quality, rigor, and depth of
theories, questions, and problems.
Currently, undergraduates prepare for entrance examinations to get in to leading
institutes such as Oxford, Cambridge, MIT, Princeton, etc. Millions of students all
over the world participate in Physics Olympiads, both national and international,
and other international physics examinations such as Physics GRE etc. In India,
millions of students take national level examinations for the prestigious Indian
Institutes of Technology (IITs) through a tough Joint Entrance Examination called
IIT-JEE with a 0.5% success rate. There are no books available on the market that
are balanced in terms of the problems and theories covered, and the quality and
quantity of the problems, for such students.
In this series, I have made a sincere attempt to provide balanced books with
theories, examples, and exercises, and with a systematic approach to concept
building. By the virtue of my experience and expertise, suggestions from and
association with my colleagues and hundreds of gifted students, this series of books
is the outcome of my sincere effort to transform physics-phobia into a physics-loving
attitude for students.

Readership
This series of books is a mixture of important, creative, and brain-storming
problems arranged systematically (by sub-topic), so that even an average student
can easily refer to the books. Detailed solutions are provided for each problem. Each
book of this series has different types of problems with a gradual increase in the
standard and difficulty level. Each problem is multi-conceptual, with a variety of
small questions. These books could be ideal for an aspirant hoping to master the
concepts of physics. In this sense, it would be highly suitable for students

xii
Problems and Solutions in Rotational Mechanics

(undergraduates) preparing for national and international Physics Olympiads as it


contains 50 years of national and international Olympiad problems in modified and
general forms. For students preparing for PhD examinations for top universities and
physics GRE examinations, this book could also be useful. In India, students
preparing for UPSC examinations and semester examinations, and physics majors in
BSc or BEng (of any branch of engineering), these books could be useful to an
extent, as handbooks of physics problems.
The entire series will ultimately be useful for millions of Indian students preparing
for JEE (mains and advanced). Any IIT-JEE student could be well-versed with the
concepts of physics with a strong analytical skill in problem-solving, even after
completing only 60%–70% of this book. Eventually, he or she could achieve JEE-
Advanced (which students can take after passing JEE-Mains) with good ranks.
To educate their students, teachers need to be well-versed in physics and deliver in
a stipulated time frame. That is possible only when we have suitable books with
scholarly set questions arranged systematically. Thus, keeping in mind all the merits
of my book series, I am hopeful that this series could become popular reference book
for thousands of potential JEE educators in India. I received unprecedented support
and recognition from educators in India for my earlier published book GRB
Understanding Physics, because it contained original problems in systematic way.
These books are out of print but still available from many e-commerce sites.
However, in this series I have resolved some of the limitations of my previous
publication, and it is balanced in many respects and hence unmatched by other
available books.

The series consists of six individual books, providing almost an entire physics
course. Each will contain nearly 400 problems with examples and exercises. This
series presents an unprecedented effort to provide the maximum variety of problems
arranged systematically according to the common syllabus of reputed undergrad-
uate physics books such as Halliday and Resnick’s Principles of Physics, Physics for
Scientists and Engineers by R A Serway and R J Beichner, University Physics
(known as ‘Sears & Zemansky’) by H Young and R Freedman, Physics by H C
Ohanian, etc. The combination of my experience and expertise over three decade of
teaching and writing, along with the contents of hundreds of books I have followed,
will form the basis for this series of books.

The common features of the series


I have had a long standing desire to prepare a set of problem books/booklets by
dividing physics into these smaller segments. This will provide almost all varieties of
problems for students in a particular small section of physics, and the students need
not purchase a complete physics book, only those for a specific area of physics.
I have handpicked many problems from various sources. Each problem has been
observed being asked in different ways in various examinations all over the world. I
analyzed each problem’s most original version and then put it in my series.
Furthermore, I prefer to put all possible questions linked with the original problem

xiii
Problems and Solutions in Rotational Mechanics

in one place, either as separate problems or as sub-questions within the original


problem.
Each volume features a mixture of small, tricky, conceptual, introductory,
complex, and lengthy brain teasing problems. Together these problems span 70%–
80% of the first year of any undergraduate course of physics or engineering in US,
UK, and Indian universities. All the questions would ultimately be useful for
national and international Physics Olympiad examinations. This book covers 70%
of the physics GRE and 50% of the PhD qualifying examinations for leading US
universities.
Each book is made to be student-friendly, balanced with theory and problems.
The quality and quantity of the content is balanced with systematic placement of the
concepts, immediate examples under each concept (the heading/sub-heading of each
chapter), scholarly set problems under each section, with a detailed solution
immediately after each problem. The main request of most students is to access a
large number of quality problems in a book arranged in a logical sequence. Thus,
these books could act as an ‘all-in-one’ resource for aspirants of various entrance
examinations fulfilling the above conditions.
All the problems can be solved using simple calculus, algebra, trigonometry, and
geometry. Therefore, in the global market these books could be regarded as an
unprecedented venture to present the entire physics curriculum with adequate
examples and problems in a student-friendly way.

About this book


Rotational mechanics is a rank-deciding area in examinations for aspiring students.
Due to its complexity average students have a fear of this area. Thus I have tried to
present the subject matter in an interesting way without the use of higher
mathematics such as Langrangian, Hamiltonian, etc. I present the examples and
problems in a proper sequence following the theories. Rotational mechanics is the
most challenging area in mechanics because it needs many concepts to solve a single
problem. The correct solution of a single problem in this area will greatly improve
the ranking of a student. Thus this portion is a rank-deciding one. Thus each
aspiring student seeks an appropriate book of rotational mechanics that could fulfill
all the criteria, but to my knowledge there is no such single book. Thus I hope this
book could be of unique value for those students.
This book has five chapters:
1. Kinematics of rotation
2. Torque and angular momentum for a point mass
3. The statics and dynamics of rigid bodies
4. The energy of rigid bodies
5. Impulse and momentum of rigid bodies.

The first chapter discusses the angular quantities, such as angular displacement,
angular velocity, and angular acceleration, the kinematics of rolling, and the
instantaneous axis of rotation. The second chapter establishes two key concepts,

xiv
Problems and Solutions in Rotational Mechanics

namely torque and angular momentum, for the motion of a particle. The third
chapter extends the application of torque and angular momentum for a system of
particles and then for rigid bodies. The fourth chapter deals with the energy aspect of
a rigid body, such as translational and rotational kinetic energy. The application of
the work–energy theorem and energy conservation are described in this chapter. The
last chapter deals with the impulse–momentum equations applied for a rigid body.
The principle of conservation of angular momentum is explained with many
interesting examples including the collision of rigid bodies.

How to use this book


It is my sincere suggestion to students to first complete the given theory with the
examples to obtain proper understanding. Then, they should try to solve the
problems in their own way. Sometimes students obtain the answers better following
their own methods, so students should not be biased by my approach. If a student
falters after repeated attempts, then they can refer to the given solutions. Due to the
creativity and modification of many of the questions, I cannot proclaim that the
book is error-free, in the first edition. However, I have tried to make it error-less. To
make the book error-free, I request the readers to critically analyze my work and
point out any mistake in the book which will be rectified as soon as possible. I will be
grateful to all my readers for their comments and suggestions for the overall
improvement of this book.

xv
Acknowledgements

It goes without saying that a teacher is worth millions of books. An ideal teacher is
built by their association with students. I acquired my experience and expertise
through association with gifted students at premier institutes. Thus, first of all, I
would like to express my gratitude to the directors of all the institutes at which I
worked. Most illustrious are Brilliant Tutorials Private Limited (BT), Madras
(Chennai), FIIT-JEE Ltd, New Delhi, and the Narayana Group of Educational
Institutions, Hyderabad.
I am indebted to the late Professor P C Samal for introducing me to the world of
education at his institute in 1988. Within a couple of years my students, such as Er
Premananda Acharya (now a senior computer scientist and physics educator) in
Bangalore, were instrumental in enhancing my problem-solving skills and setting up
my own institute comprising outstanding students at that time. The clarity of the
concepts and the style of approach towards presentations and problem-solving has
been inherited from some of my venerable professors, such as Professor P K Dhir
and Professor Sarat Mahapatra for electromagnetism, Professor Jatin Dash for
general physics, and Professor Kulamani Samal for modern physics, who taught me
to present modern physics with a historical evidence. All the attributes of my gifted
teachers (many of whom have left their mortal bodies) and students (who are now
global leaders in their own fields) are reflected in all the volumes of my problem
books to be published by IOP Publishing, in addition to the current volume. Thus I
express my sincere thanks to all my revered teachers (gurus) and past students.
I am grateful to Professor T Surya Kumar for sharing his problem-solving skills
while I was actively involved in the Doubt Letter Cell (department engaged in
clearing physics doubts in the form of letters sent by students across the country) in
BT from 1004 to 1999. He discovered a talent for writing in me and inspired me to
write in addition to teaching. He edited a collection of creative problems in
mechanics, framed by me. I am also grateful to the principal Professor M S
Subramanian in BT, Madras, who motivated me to handle the classes and doubt
letters most effectively. This helped me to dispel the most crucial doubts in
mechanics and was ultimately helpful in preparing the manuscript of this book
after two and half decades. My personal contact with the late Professor G N
Subramanian of BT, who was well-versed in both physics and mathematics, helped
me greatly in understanding some intricacies of mechanics. He was my senior
colleague at BT and Narayana Group, Hyderabad.
I extend my sincere thanks to Professor Bhanumati at BT for encouraging me to
further my professional career at FIIT-JEE Limited, New Delhi. I thank my Head
of Department Er Srikant Kumar and other seniors Er P K Mishra and Er Nagrath,
etc, for imparting the best possible teacher training for my physics knowledge
enhancement program at FIIT-JEE between 1999 and 2001. It has had a great
positive impact on this current series of problem books.
At FIIE-JEE, I am grateful to my colleagues Er B M Sharma (author of the
Cengage publication), Er D C Pandey (author of the Arihant publication),

xvi
Problems and Solutions in Rotational Mechanics

Mr M Husain (now heading the physics department of a leading chain of schools),


Er Sunil Deshpande (author and owner of his institute), Mr M Pallai, and
Mr S Rout (now owner of Aryabhatta Institute, Berhampur, Odisha) for their
kind cooperation in completing the highest selling study material for tough
competitive examinations, popularly known as Rankers Study Material (RSM), in
2001. After three years, I started writing five volumes of a book with the title GRB
Understanding Physics. I am grateful to the late Mr Prakash Chand Bathla for
offering me the opportunity to write these books under his nationally leading
publication house (G R Bathla and Sons). Based upon my experience as a national
level author and educator, I was able to attain my present status of working under an
international publishing house (IOP Publishing). I am grateful to the senior scientist
and educator Mr S M Pathak for his promise to edit the books to make them error-
free in their future editions.
I would like to thank all my previous top students who edited my books while
I headed the Department of Physics at the Narayana Group of educational
institutions. Furthermore, I express my deepest gratitude to the principals and
deans of Nayayana Group, Hyderabad, in particular Mr Krishna Reddy and
Mr Ramalinga Reddy, with whom I worked a major portion of my professional
career, for giving me operational freedom, status, stability, and respect. There, I was
able to complete some theories and examples of the current series in rudimentary
form. In Hyderabad, leading educators such as Mr Aditya Sachan, Er L N Prusty,
Er Sekhar Somnath, and Mr Monoj Pandey were constant motivators of my writing
skills. Another leading physics educator Mr S K Singh of Chaitanya Educational
Group was a good critic of my work and spent some time editing my manuscripts.
I am also grateful to Professor Kundal Rao of Narayana IIT and Professor
Srinivasa Chary of Sri-Mega for their suggestions and inspiration for my published
works.
Thus, I have been directly and indirectly inspired by many people. Primarily,
I would like to express my profound gratitude to my wife Usha for supporting me
and bearing with me during the pandemic in 2020 when I started conceptualizing
this book. As this project is comprised of six books covering the entire field of
physics, it has been an arduous task, thus it required full-time effort to complete it in
time. It was my long standing dream to deliver these books for a global audience as
an output of my vast experience and expertise gained in the afore-mentioned ways.
I had to honestly decline lucrative offers from reputed institutions in order to
complete this project as soon as possible in best possible way. Although it depleted
my financial position and my wife was not doing well in the pandemic, I preferred to
occupy the side of propagating knowledge rather than money-making, because
knowledge is far more valuable than money. The authors of great books such as
Problems in Physics by I E Irodov and Concepts of Physics by H C Verma are
brilliant examples of advocates of knowledge over the vanities of the world.
Therefore, I offer my humble obeisance to these professors who inspired me to
write books to impart knowledge to millions of students.
I remain obliged to the commissioning editor of IOP Publishing Mr John Navas
for his insightful comments, suggestions, and expertise, which have enhanced the

xvii
Problems and Solutions in Rotational Mechanics

rigor and depth of this work. He took special interest in getting my book proposal
reviewed by the appropriate experts who could sense something interestingly and
useful for students and provided their positive feedback. Unfortunately, after signing
the contract, he left IOP Publishing. Then Ms Phoebe Hooper was put in charge of
my book project and Mr David McDade replaced Mr John Navas. These two new
people took some time to streamline the work process due to the abrupt change in
management. I thank both of them for their best efforts in handling the publication
work of my book.
I express my gratitude to my ex-publisher Mr Monoj Bathla who recommended
me to accept the offer from IOP Publishing after realizing the suitability of the
publisher for my work. As I taught thousands of students in last two decades, it is
not possible to mention the name of each student who reviewed my original
manuscript again and again, and I am grateful to each one. Furthermore, I am
grateful to the leading IIT-JEE faculty members of India and peers who have
provided valuable feedback, stimulating discussions, and intellectual support during
the preparation of this book. Some of them are worth noting: Er Anurag Mishra (the
author of Physics) and Er Ashish Arora (the author of Physics Galaxy), both from
Allen Career Institute, Kota, who are admirers of my previous books. The personal
motivation and expectations of these top-notch physics educators were instrumental
in making this problem book series remarkable.
Additionally, I would like to acknowledge the professional and financial support
provided by IOP Publishing, which not only alleviated my financial hardship to an
extent but also enabled me to deliver this project in time.
Four typists prepared my manuscripts at different places and times. I sincerely
thank Mr Bismay Parida (Readers Institute, Balasore, Odisha State) for his contin-
uous effort in typing the manuscripts in time. Furthermore, Mr Sai, Mr Venkatesh and
Mr Bharat (from Hyderabad, Telangana State) typed a considerable portion of the
manuscript of the problems sections of the entire series.
Finally, I am deeply grateful to my family and friends for their unwavering love,
encouragement, and understanding throughout this challenging yet fulfilling pub-
lishing journey.
Recapitulating, the completion of this series of problem books will be the
culmination of an arduous yet rewarding journey, and I owe my deepest gratitude
to the afore-mentioned individuals and organizations who have supported me all
along my career. Finally, I again express my profound appreciation to all my
directors, principals, deans, seniors, colleagues, students, and teachers all over the
country for their unwavering guidance, encouragement, and scholarly mentorship
throughout the entire preparation process of the series. Their expertise, patience, and
constructive feedback have been invaluable in shaping the direction and quality of
this work in my physics education research in the mode of active teaching and active
learning (ATAL), which was coined by Mr S K Singh to whom I am deeply
indebted.
As this project of six books has been a huge task, I had to take leave as an
extramural researcher from Sofia University, Bulgaria. I am grateful to the
university for granting leave of one year. Furthermore, I express my sincere

xviii
Problems and Solutions in Rotational Mechanics

gratitude for the thoughtful suggestion by my professor Dr Alexander Gungov to


support my book publication with ongoing research at the university.
I have many friends and professors in the UK who encouraged me to write these
books. As an alumnus of the Universities of Oxford and Strathclyde I have deep
respect for all my professors, in particular Professor Peter Dobson (Oxford
University) who taught me how to do things to perfection and I am applying this
idea in the present publication. I am deeply indebted to Dr Benjamin Hourahine and
Professor Yu Chen of the University of Strathclyde for imparting a standard
knowledge of nano-science so that I could include this fast growing field in my
problem book series.
One of my notable friends is Mr Rajinder Sehra, director of S&RJ Ltd and Foot
Print Media Production near Glasgow. His constant encouragement in writing this
series is also praiseworthy. I will remain grateful to him for his moral and physical
support during my stay in Glasgow and constant encouragement for my book
writing. The admiration from my professors at Strathclyde University and Strath—
student union representative James Higgins, two times Strathclyde University
Student Union president Mr Matt Crilly, Gary Paterson (Ex-Vice president of
Starth Union) and Gerry McDonnell (Ex-President of Mature Student Association
and Ex-Vice president, Strath Union, Strathclyde University toward my five
volumes of physics books kept in the library of the Mature Student Association
of the University of Strathclyde has also been a great source of my motivation for
this current series. I express my deepest gratitude to them.
Last but not least, I am wholeheartedly grateful to Mr Himanshu Sekhar Dey,
one of the directors of Readers Institute, Balasore, for taking the greatest portion of
the burden of the institute and freeing me to devote myself entirely to this project.
This book would not have been possible without the collective contributions and
support of all those mentioned above. Their guidance and encouragement have been
instrumental in the completion of this significant milestone in my publishing career.
Taking this as a blessing of the Almighty, I pray for the attainment of knowledge
that would be an ultimate solution to all problems of human beings and other living
entities.

Pradeep Kumar Sharma


19 June 2024

xix
Author biography

Pradeep Kumar Sharma


He is a well-known physics educator in India possessing more than three decades of
experience in physics education and research in training the aspirants of the joint
entrance examination conducted by prestigious Indian Institutes of Technology,
popularly known as IIT-JEE. He has trained thousands of students for under-
graduate entrance examinations such as IIT-JEE etc. Many of his students also
qualified in national and international physics Olympiads. His vast experience as a
potential teacher, team leader and head of the department in some premier institutes
like Brilliant tutorials, (Chennai), FIIT-JEE Ltd (New Delhi), Narayana Group
(Andhra and Telangana) etc, made him extend his service as a consultant physicist
to mentor both students and teachers of reputed groups in India.
He has authored bestselling study materials, five books known as GRB
Understanding Physics for the entrance examinations. He has been associating as
a research scholar of physics education, nanoscience, metaphysics and management
in some Indian and foreign universities such as Oxford University, Strathclyde
University, Sofia University and Indian Institute of Technology, Patna.
Furthermore, he is continuing his research while affiliated with various national
and international organizations such as IEEE (USA), IET (UK), IE(I), IOP(UK)
etc. He has published dozens of papers in national and international journals like
IEEE-Scopus journals and journals published by Institute of Physics (UK). He is
currently busy in completing the problems and solutions of a series of six books
which will be ready to publish very shortly.
At present, he is actively involved with a team of top-notch educators, to design a
new method of interactive education called Active Teaching and Active Learning
(ATAL) that will make things easy for an average student to learn physics.

xx
IOP Publishing

Problems and Solutions in Rotational Mechanics


Pradeep Kumar Sharma

Chapter 1
Kinematics of rotation

1.1 Definition of a rigid body


A body is said to be ideally rigid when its deformation (elongation, compression,
and twisting) under the application of forces and torques is zero. However, nothing
is perfectly rigid. If the deformation of the body is negligible compared to its size,
you can call it a practically rigid body. So the shape and size of a rigid body remain
practically constant. For this to happen, the distance rAB between any two points of
a rigid body remains practically constant. This means that,
drAB
= 0.
dt

• The component of relative velocity vAB between these two points along the
line AB will be zero.
• Then, A seems to move perpendicular to AB relative to B.
• Similarly, B seems to move perpendicular to AB relative to A.

doi:10.1088/978-0-7503-6472-0ch1 1-1 ª IOP Publishing Ltd 2024. All rights,


including for text and data mining (TDM), artificial intelligence (AI) training, and similar technologies, are reserved.
Problems and Solutions in Rotational Mechanics

Example 1 A rod AB is kept against a vertical wall making an angle of 30° with
vertical as shown in the figure. If the end A of the rod moves with a velocity of
vA = 5 m s−1 downwards, find the linear velocity of its end B.

Solution
Since the rod is rigid, the component of vAB along the rod is zero. This means we
can equate the velocities of the end points A and B along the length of the rod.
So, 5 cos 30° = vB cos 60°. This gives
vB = 5 3 ms−1.

1.2 Translation of a rigid body


Let us consider a moving rectangular plate and draw a straight line AB on it. Take a
fixed straight line CD outside the plate. Let AB make an angle q with the reference line
CD. If the straight line AB does not change its angle of orientation q relative to the
fixed reference straight line CD when the plate moves, the plate is said to be translating
without any rotation. This is known as pure translation or simply translation.

1-2
Problems and Solutions in Rotational Mechanics

In pure translation, all points of the body:


• undergo equal displacements.
• have the same velocity.
• have the same acceleration.

This means that the relative motion (that is, relative displacement, relative velocity, and
relative acceleration) between any two points of the rigid body is zero in pure translation.
There are two types of translation, namely rectilinear translation and curvilinear
translation:
• In rectilinear translation any point of the rigid body moves in a straight line.
• In curvilinear translation any point of the rigid body moves in a curve.

1.3 Angular motion or rotation of a rigid body


For the sake of simplicity, let us consider a thin plate (a planar rigid body). Draw a
line AB on the plate. We can see in the following figure that the line changes its angle
of its orientation from q1 to q2 relative to a fixed reference line CD. Then the body is
said to be rotating (or undergoing an angular motion).

1-3
Problems and Solutions in Rotational Mechanics

1.4 Angular velocity of a rigid body


Draw two lines 1 and 2 on the plate. Let these lines make angles q1 and q2 ,
respectively, with a horizontal (x-axis) as shown in the figure. Since the lines are fixed
with the rigid body, the angle b between the lines does not change. So ddbt = 0.
dq1 dq2
Putting b = (q1 − q2 ) and differentiating both sides we have dt
− dt
= 0. This
dq1 dq2
gives dt
= dt
.
This states that, at a given instant, all lines drawn inside the plate
change their angle of orientation with respect to a given reference line at the same
rate. This is known as the angular velocity of the rigid body.

1.5 Calculation of the angular velocity


1.5.1 Method 1: ground frame method
Draw a line on the rigid body. Let its angle of inclination be q with the x- or y-axis.
Express either the x- or y-coordinate of any point of the rigid body in terms of q .
Then differentiating both sides find w = ddqt when dx or dy is given. Hence the angular
dt dt
velocity of a rigid body is equal to the rate of change of the angle made by a straight
line drawn inside the rigid body with any fixed reference line. The formula of angular
velocity is w = ddqt . If q increases with time, the sense of w is in the increasing order of
q and vice versa.

1-4
Problems and Solutions in Rotational Mechanics

1.5.2 Method 2: relative motion method


In this method first of all we consider two points (A and B, say) of the rigid body.
Since the distance between these points is r, the component of relative velocity
between A and B along the line AB is equal to zero, as discussed in section 1.1. Then
A and B seem to move relative to each other perpendicular to the line AB in opposite
directions. This means
 
vAB = − v BA .

However, A and B seem to turn relative to each other in the same sense (in a
clockwise manner, as shown in the figure) with the same magnitude of angular
velocity. In other words, their relative angular velocities are exactly equal to each
other (in both magnitude and direction). So we can write
   
wAB = w BA = w rel = w ,
v
where w = rel r
is known as the angular velocity of the rigid body and
vrel = vAB = vBA .

Hence in this method:


• The angular velocity of a rigid body is equal to the relative angular velocity
between any two points of the rigid body.
• The relative velocities between any two points of a rotating rigid body are
negative vectors (equal in magnitude and oppositely directed).
• The relative angular velocities between any two points of a rotating rigid
body are equal vectors (equal in both magnitude and direction).
• At a given instant of time, the relative angular velocities between all possible
pairs of points of a rotating rigid body are equal, which is defined as the angular
velocity of the rigid body, denoted as w . Hence, the angular velocity of a rigid
body does not depend upon any reference point or reference frame we choose.
     
• The vector relation between vAB, rAB, and wAB can be given as wAB × rAB = vAB.

Example 2 A rod AB of length l is leaning against the wall making an angle q with
the wall. If the top end A of the rod moves with a downward velocity u, find the
angular velocity of the rod.

1-5
Problems and Solutions in Rotational Mechanics

Solution
Method 1 (ground frame method)
The position of the point A of the rod is given as y = l cos q . Differentiating both
sides with time, we have ddyt = −l sin q ddqt . Put ddyt = −u (as y decreases) and ddqt = w
(because q increases) as the bottom moves towards the right. Then we have
w = l sin q . The positive value of w means physically an anti-clockwise sense of
u

rotation of the rod.

Method 2 (relative motion method)


Let the velocity of the bottom end of the rod be v. By equating the components of
velocities of the ends of the rod (along the rod), we have v sin q = u cos q .
So v = u cot q . The relative velocity between the ends is vrel = u sin q + v cos q ,
where v = u cot q . Then, simplifying the terms, we can find the same result
u
w = vrel /l = l sin q
.

1.6 Angular acceleration and its calculation


1.6.1 The ground frame method
We define the linear acceleration as the rate of change in linear velocity given by the
formula v = dx/dt. Similarly, we define the angular acceleration of a rigid body as
the rate of change of angular velocity of the rigid body given as

1-6
Problems and Solutions in Rotational Mechanics

dw dw
a= =w .
dt dq
dq dw
Since w = dt
and a = dt
we can write

dw d2q
a= = 2.
dt dt

This expression of angular acceleration is given based on the ground frame method.
However, in the relative motion method we can also determine the angular
acceleration, as follows.

1.6.2 Relative motion method


If you stand at the point B and look at any point A on the rigid body (thin plate) you
can see that A is moving in a circular path with a velocity vAB. As the distance
between A and B does not change, the radius of the circular path is rAB = r, say. Let
the tangential and radial or normal acceleration of A observed by (or relative to B)
 
be a t and a n , respectively. By applying the kinematics of circular motion of A
relative to B, we will obtain the following equations:

dvAB
a n = rw 2, at = .
dt

We define angular acceleration as the rate of change in angular velocity. In the


relative motion method, the angular velocity of the rigid body is given:

w BA = w rel = w = v BA / r.

So the angular acceleration of the rigid body is given as

a=
dw
=
d BA
r ( ) = d( v
v
BA )
.
dt dt r dt
d(vBA )
Putting r dt
= a t , we have
at
a= .
r
We can use the following vector equations of the tangential and normal accel-
erations of a point relative to any other point of the rigid body borrowed from the
kinematics of circular motion:
     
at = a × r and a n = w × v BA .

1-7
Problems and Solutions in Rotational Mechanics

So the net acceleration of A relative to B can be given by adding the above two
components:
  
aAB = at + a n .

Example 3 In example 2, if the velocity u of the end A of the rod remains constant,
find (a) the angular acceleration of the rod and (b) the linear acceleration of the end
B of the rod.
Solution

(a) The angular acceleration is given as


dw u
a= , where w = .
dt l sin q
Then we have
d(cosec q ) dq
a=u = − (u / l ) cosec q cot q⎛ ⎞ .
l dt ⎝ dt ⎠
dq u
Putting the value of dt
=w= l sin q
and simplifying the factors, we have

a = − (u / l )2 cosec 2 q cot q .
Since the answer is a positive quantity, the direction (sense) of a is the same
as that of the angle q which increases in an anti-clockwise sense.
(b) From example 2 the velocity of B is given as
v = u cot q .
So the acceleration of B is
dv d(u cot q )
a′ = = .
dt dt

1-8
Problems and Solutions in Rotational Mechanics

So a′ = −u cosec 2q(dq /dt ).


Putting the value of ddqt = w = u
l cos q
and simplifying the factors, we have

u2
a′ = − sec q cosec 2 q .
l
Negative value signifies the point B is retarding.

1.6.3 Gyroscopic motion


In planar motion, the directions of the angular velocity and angular acceleration
are axial, which is perpendicular to the plane of the rigid body. However, in three-
dimensional rotational motion (gyroscopic motion), the direction of angular
velocity also changes. Thus another component of angular acceleration comes
into play due to the change in the direction of the angular velocity, which can be
given as
a = ww ′,
where w is the angular velocity of the rigid body (cylinder) and w′ is the angular
velocity of the axis of rotation of the rigid body (cylinder). You can refer to Problem
28 for its derivation.

1.7 Fixed axis rotation


This is the rotation of a rigid body around a fixed axis. The meaning of fixed axis is
that the axis does not move. A rotating ceiling fan, the rotation of the hands of a
wall clock, and the rotating blades of a mixer are few examples of the fixed axis
rotation.
In fixed axis rotation each point moves in a circular path about the axis of
rotation. If we take a point P from the axis O, so that PO = r, the velocity of P is
given as v = rw . The tangential and radial acceleration of P can be given as
at = dv/dt and ar = v2/r = rw 2 , respectively, by applying the kinematics of circular
motion.

1-9
Problems and Solutions in Rotational Mechanics

Fixed axis rotation is a special case of rotation where we can directly apply the
kinematics of circular motion for each point of the body. There are two types of
fixed axis rotation, as follows.

1.7.1 Centroidal
Let the uniform disc be pivoted with a fixed point at its center. As its axis of rotation
is fixed and it passes through the center of mass of the rigid body, this fixed axis
rotation is centroidal. Hence, in the centroidal fixed axis rotation the center of mass
does not move. This type of rotation is called pure rotation.

1.7.2 Non-centroidal
If the uniform disc is pivoted at its end P, this means the axis of rotation does not
pass through the center of mass C of the rod. Thus this fixed axis rotation is non-
centroidal.

The velocity of any point of the rigid body in fixed axis rotation varies linearly with
radial distance which is given as v = rw .

1-10
Problems and Solutions in Rotational Mechanics

Example 4 In the given figure a disc rotates with a constant angular velocity about a
vertical axis passing through the point P, the maximum speeds and accelerations of
the points C and Q of the disc are u, v and a, A. respectively. Find the value of (a) v/u
and (b) a/A. Assume that in each case the angular acceleration is zero and the
angular velocities are equal.

Solution

(a) Since the angular acceleration is zero, all points of the discs execute uniform
circular motion. So the speeds of C and Q are, respectively, given as u = Rw
and v = 2Rw . Then v/u = 2.
(b) For uniform circular motion, the total acceleration is centripetal. Then, the
ratio of centripetal accelerations of C and Q are given as
a / A = (Rw 2) / (2Rw 2) = 1 / 2.

1.8 Combined motion


It is a combination of pure translation and pure rotation. Both phenomena occur
simultaneously. In pure rotation, the center of mass remains at rest and the body
spins about the center of mass axis. In pure translation, the body does not rotate and
hence all points move with the same velocity v . If we combine both phenomena
(pure translation and pure rotation), we can see that the center of mass has a certain
velocity v and the body spins with an angular velocity w , as shown in the figure.

1-11
Problems and Solutions in Rotational Mechanics

Examples of combined motion are the rotating blades of a flying helicopter, a drone,
rolling bodies, and the orbiting of the spinning earth, etc.

1.9 Finding the velocity and acceleration of a point of a rigid body


under combined motion
Let a lamina (plate) undergo a combined motion with the angular velocity and
angular acceleration w and a, respectively. Let us chose two points A and B on the
plate. If the velocity and acceleration of A are given as vA and aA, respectively, how
do we find the velocity and acceleration of the other point B? Let us see.

Velocity of A: The velocity of A is given as


  
vA = vAB + v B ,
  
where vAB = w × rAB.

The magnitude of vAB is given as
vAB = rAB w .
Acceleration of A: The acceleration of A relative to ground is given as
  
aA = aAB + a B,

where aAB is equal to the vector sum of its tangential and radial components, given
as
  
aAB = at + a r .
Since the point A moves in a circle relative to B, the radial and tangential
accelerations of B relative to A are, respectively, given as follows:
  
at = a × rAB,
  
a r = w × vAB .

1-12
Problems and Solutions in Rotational Mechanics

Their magnitudes are given as


at = rAB a and a r = rw 2.

Example 5 A uniform disc of radius R lying in the x–y plane spins with a constant
angular velocity w in anticlockwise sense. The center C of the disc moves with a
velocity v. Find the velocities of the points A, B, D, and E of the disc.

Solution
The velocity of A is given as
  
vA = vAC + vC
= + Rwiˆ + viˆ = (v + Rw )iˆ.
The velocity of B is given as
  
v B = v BC + vC .
= − Rwjˆ + viˆ
= viˆ − Rwjˆ .
The velocity of D is given as
  
v D = v DC + vC .
= − Rwiˆ + viˆ = (v − Rw )iˆ.
The velocity of E is given as
  
v E = v EC + vC .
= Rwjˆ + viˆ
= viˆ + Rwjˆ .

1-13
Problems and Solutions in Rotational Mechanics

Example 6 A uniform disc of radius R lying in the x–y plane spins with a constant
 
angular velocity w = wk̂ . The center of the disc moves with an acceleration a C = aiˆ .
Find the accelerations of the points A, B, D, and E of the disc.

Solution
The acceleration of A is given as
  
aA = aAC + a C
= Rw 2jˆ + aiˆ
= aiˆ + Rw 2jˆ .
The acceleration of B is given as
  
a B = a BC + a C
= Rw 2iˆ + aiˆ
= (a + Rw 2)iˆ.

1-14
Problems and Solutions in Rotational Mechanics

The acceleration of D is given as


  
a D = a DC + a C
= Rw 2jˆ + aiˆ
= aiˆ − Rw 2jˆ .
The acceleration of E is given as
  
aE = aEC + a C
= Rw 2iˆ + aiˆ
= (a − Rw 2)iˆ.


Example 7 In the previous example if we give an angular acceleration a = ak̂ to the
disc: (a) draw each component of acceleration at A, B, D and E, and (b) find the
acceleration at the given points.
Solution

(a) The additional acceleration at comes from the angular acceleration a whose
magnitude is given as a t = ra and the direction is tangential to the disc, as
shown in the figure.

1-15
Problems and Solutions in Rotational Mechanics

(b) The accelerations at the given points are given as follows:



aA = (at + a C )iˆ + a r jˆ ,

a B = (a C + a r )iˆ − at jˆ ,

a D = (a C − at )iˆ − a r jˆ ,

aE = (a C − a r )iˆ + at jˆ ,
where a t = Ra and
a r = Rw 2 and a C = a .

1.10 The kinematics of rolling


This is the combined motion of a body such as a disc, sphere, etc, on a surface such
that the lowest point of the body does not slide relative to the surface in contact. This
means that the relative velocity of the contacting points between the body and
surface is zero. In the following figure the disc rolls on a horizontal surface. This
needs the condition that the lowest point A of the disc does not slide with the fixed
horizontal surface. Since the horizontal surface is stationary, the lowest point of the
disc must remain at rest instantaneously. So vA = 0. If the center of mass of the disc
moves with a velocity vC and the disc spins with an angular velocity w , referring to
the example 5, the velocity of A is given as
  
vA = vAC + vC .
= − Rwiˆ + viˆ
= (v − Rw )iˆ.
Putting vA = 0 in the above expression, we have vC = R w . Differentiating both sides
with time, we have dvC/dt = Rdw /dt . This means aC = Ra.
To summarize, for a rolling body on a fixed surface, if the velocity and
acceleration of the center of mass of the body are vC and aC, respectively, the
angular velocity w and angular acceleration a of the body can be related with vc and

ac as vC = R w and aC = Ra. In rolling on a fixed surface, the velocity and tangential
acceleration of the lowest point of the rolling body will be zero. This means that the
tangential or horizontal component of acceleration of the lowest point A is zero.

However, the net acceleration of A is nonzero: (aA)tangential = 0, but a A = Rw 2 jˆ . If
the surface moves, these expressions do not hold.

Example 8 Let a disc roll without sliding on a fixed horizontal surface with a
constant velocity. Establish the relation between the linear velocity v and angular
velocity ω of the disc in the following figures.

1-16
Problems and Solutions in Rotational Mechanics

Solution

(a) The velocity of the lowest point A is


  
vA = vAC + vC .
= − Rwiˆ + viˆ
= (v − Rw )iˆ.
For rolling, putting vA = 0, we have
(v − Rw )iˆ = 0.
Then we have
v = Rw .

(b) The velocity of the lowest point A is


  
vA = vAC + vC
= Rwiˆ + viˆ
= (v + Rw )iˆ.
For rolling, putting vA = 0, we have
(v + Rw )iˆ = 0.
Then we have
v = −Rw .

Example 9 Prove that rolling is a mixture of pure translation and pure rotation.

Solution
In pure translation, all points of the disc move with same velocity v towards the
right. In pure rotation the center of mass is fixed and the disc must spin clockwise
with an angular velocity

1-17
Problems and Solutions in Rotational Mechanics


w = - wkˆ = − v / Rkˆ (clockwise) .
Then the velocity of the lowest point A is given as
  
vA = vAC + vC
= − Rwiˆ + viˆ
= (v − Rw )iˆ.
Putting w = v/R , we have vA = 0 which is the required condition for rolling of the
disc as discussed earlier.

We cannot see these two effects (pure rotation and pure translation) separately with
the naked eye. What we can see is the combination of these two effects. When we
combine pure translation and pure rotation, the velocity distribution in the rolling
disc will be completely different from that in both pure rotation and pure transla-
tion. By combining the velocities using the formula given in section 1.9, we can find
the velocity of any point of the rolling body. We can see that the velocity at A is zero.
This means that the lowest point of the rolling body always remains at rest
instantaneously. Hence, the lowest point of a rolling body on a fixed surface is the
instantaneous center (IC) of rotation. This is also called the instantaneous axis of
rotation (IAR) as discussed in the next section.
For the sake of simplicity, the velocity distribution (profile) on the vertical diameter
of the rolling disc is split into pure translation and pure rotation. You can see that
velocity is uniform in pure translation. In pure rotation velocity changes linearly from
–v at the bottom, zero at the center and +v at the top of the disc. However, the
addition of these two effects gives rise to pure rolling in which velocity increases from
zero at the bottom to +2v at the top of the disc, as shown in the figure.
• The body is said to be rolling on a surface when the relative velocity of the
contacting points between the body and the surface is zero.
• If the surface is fixed/stationary, for rolling, the lowest point of the body does
not move instantaneously.
• The lowest point of a body rolling on a fixed surface is the IAR (please see the
next section).

1-18
Problems and Solutions in Rotational Mechanics

Example 10 Referring to the last figure, a disc of radius R rolls on the horizontal
surface along the +x-axis. The center O of the disc moves with a constant velocity

v = viˆ . Find the (a) velocities and (b) accelerations of the points A, B, C, and D of
the disc.

Solution

(a) The velocity of A is given as


  
vA = vAO + vO
= − Rwiˆ + viˆ
= (v − Rw )iˆ = 0.
So we have
Rw = v . (1.1)
The velocity of B is given as
  
v B = v BO + vO
(1.2)
= Rwjˆ + viˆ.

Putting Rw = v from equation (1.1) in equation (1.2) we have



v B = viˆ + vjˆ
= v(iˆ + jˆ ) .
The velocity of C is given as
  
vC = vCO + vO
= Rwiˆ + viˆ
= (v + v)iˆ = 2viˆ.
The velocity of D is given as
  
v D = v DO + vO
= − Rwjˆ + viˆ
= − vjˆ + viˆ = v( + iˆ − jˆ ) .

1-19
Problems and Solutions in Rotational Mechanics

(b) Differentiating both sides of equation (1.1) we have


dw dv
at = R = = 0,
dt dt
then we can write
at = ao = 0. (1.3)
So, the acceleration of A is

aA = ( − at + ao )iˆ + a r jˆ . (1.4)

Using equations (1.3) and (1.4)



aA = a r jˆ = (v 2 / R) jˆ .
The acceleration of B, C and D can be given as (please refer to example 7)

a B = (a O + a r )iˆ + at jˆ

a C = (a O + at )iˆ − a r jˆ

a D = (a O − a r )iˆ − at jˆ ,
where a t = Ra = 0 and
a r = v 2 / R and ao = 0.
Then we have

a B = (v 2 / R)iˆ

a C = − (v 2 / R)jˆ

a D = − (v 2 / R)iˆ.

Example 11 A uniform disc of radius R rolls on a horizontal surface so that its


center C moves with a constant velocity v. Find (a) the velocity and (b) the
acceleration of an arbitrary point A on the periphery of the disc.

1-20
Problems and Solutions in Rotational Mechanics

Solution

(a) Since point P moves perpendicular to the line AP with an angular velocity w , the
velocity of the point A is given as vP = (AP)w , where w = Rv as per the condition
of rolling and AP = 2R cos ∅, where ∅ = q /2. Then vP = 2v cos(q /2).
   
(b) The acceleration of point P is given as aP = aPO + a O , where a O = 0

because vO = constant. Then
 
a P = a PO.

Since aPO does not have a tangential component a t = Ra = a = 0, it must
be radially inward. So aP = Rw 2 = v2 /R (because w = Rv as per the
condition of rolling).

1.11 General kinematical misconceptions


By now you might have understood that an angular motion or rotation is not a
circular motion. In fixed axis rotation, it is true that each particle in the body moves
in a circular path relative to the ground. But, generally in combined motion, points
of a rigid body move in arbitrary curves. So the rotation of a rigid body should not
be misinterpreted as a circular motion. Circular motion is used for a particle or
point, whereas rotation is used for a rigid body (or a straight line drawn inside the
rigid body).
There is another misconception between fixed axis rotation and pure rotation.
Both are conceptually different. Non-centroidal fixed axis rotation is a combination
of translation and rotation because the center of mass moves. However, in centroidal
fixed axis rotation there is no translation as the center of mass remains fixed because
the body is pivoted at the center of mass.
The third misconception is that ‘for a rigid body to rotate it must be pivoted at a
fixed point’. If you throw a ball by giving it a spin, this a combined unconstrained
motion. The body is not pivoted anywhere but is still rotating with a moving center
of mass. You may wonder how this is possible? Then, you may ask ‘who causes the
rotation?’ which will be discussed in chapter 3. So let us accept the fact (without

1-21
Problems and Solutions in Rotational Mechanics

knowing its cause) that a rigid body can rotate independently, possessing angular
velocity and angular acceleration without being pivoted at a fixed support.
The fourth important fact which is often misinterpreted is that angular quantities
(displacement, velocity, and acceleration) are the internal properties (or phenomena)
of the body but not the properties of any point of the rigid body relative to outside
points.

1.12 Kinematical equations for uniform angular acceleration


Then, for the sake of broader understanding, let us consider a spinning disc
undergoing a general or combined motion. Let, at time t = 0, the velocity of its
center of mass C be vo and the angular velocity of the body be wo. After a time t, let
the velocity of the center of mass be v and the angular velocity of the disc be w , as
shown in the figure. Then, the general kinematical equations for rotation and
translation of the disc are given as follows.

For rotation we have


w = dq / dt .
By cross-multiplying, the angular displacement of the disc during an elementary
time dt is
dq = w dt .
Integrating both sides, the net angular displacement is
t
q= ∫0 w dt (1.5)

The angular acceleration of the disc is given as


dw dw
a= and a = w .
dt dq
If we assume that the direction of angular acceleration is opposite to that of the
angular velocity, we just impose a negative sign on the right-hand side of the
equations. For a small time interval dt the angular velocity is given as
dw = a dt .

1-22
Problems and Solutions in Rotational Mechanics

It is given that at time t = 0, w = wo, and at time t = t, w = w . So, integrating both


sides, we have
w t

∫w o
dw = ∫0 a dt .

If you assume a constant angular acceleration of the disc, just take a out of the
integral to obtain
w t

∫w o
dw = a ∫0 dt .

After evaluating the integral, we have


w = wo + at. (1.6)
Putting w from equation (1.6) in equation (1.5) we have
t
q= ∫0 (wo + at )dt .

Evaluating the integral, finally we have


1 2
q = wo t + at .
2
From the general equation a = w ddwq it can be written in the differential form as

dq = w dw .
After undergoing an angular displacement q the angular velocity of the disc changes
from wo to w . After integrating both sides of the last expression, we have
q w

∫0 dq = ∫w o
w dw .

Evaluating the integral we have


w 2 − w 20 = 2aq . (1.7)

Example 12 A ceiling fan rotates with a frequency of 300 revolutions per minute
(RPMs). If it is switched off at t = 0, it slows down with a constant angular
retardation. If it stops after a minute, find (a) the angular acceleration, (b) the
average angular speed, and (c) the total (i) angular distance covered and (ii) number
of rotations until the fan stops.
Solution

(a) The angular acceleration is


a = (w − wo ) / t.
Put w = 0 as it stops after t = 60 s and wo = 2p f = 2(22/7)(300/60) = (220/7)
rad s−1 in the above equation to obtain

1-23
Problems and Solutions in Rotational Mechanics

a = − 11 / 21 rad s−2 .
(b) The formula for the average speed is
wav = (w + wo ) / 2,
where w = 0 and wo = (220/7) rad s−1. This yields
wav = (110 / 7) rad s−1.

(c)
(i) The total angular distance covered by the fan is given as
110 ⎞
q = wav t = ⎛ (60) = 6600 / 7 rad.
⎝ 7 ⎠

(ii) The total number of rotation of the fan is


6600
q
N= = 7
= 150.
2p 2× ( )
22
7

1.13 The instantaneous axis of rotation (IAR)


Inside a rotating rigid body, there exists a point either inside the physical body (or
imaginary extended body) that does not move at a given instant. This is called the
instantaneous axis (center) of rotation.

1.13.1 Calculation of the position of the IAR


(i) vP and w are given:
Let a planer rigid body perform a combined motion with an angular
velocity w . If the velocity of a point Q of the rigid body is given as vQ, how
do we find the location of IAR? Let us see.
Let us assume that O is the IAR of the rigid body located at a distance x
from the P whose velocity is given as v. As per the definition the velocity of
the IAR must be zero at the given instant. So, applying the addition of
velocities, the velocity of the IAR at O is
  
vO = vOP + v P ,
 
where vOP = −xwiˆ and vP = viˆ (given). Then, we have
x = v /w .
Otherwise: We can think of the point O as instantaneously (but not
permanently) at rest. So each point of the body (the point P, say) turns
around the point O with the same angular velocity w = v/x , because vP is
perpendicular to OP. So x = wv .

1-24
Problems and Solutions in Rotational Mechanics

Example 14 The top end A of a rod of length is given a velocity v and


simultaneously the rod is given an anti-clockwise spin w = 2vl , as shown in the
figure. Locate the IAR.

Solution
Let the IAR (point P) be located at a distance r from the end A of the rod. As the
   
point P does not move at the given instant, vP = vPA + vA = 0, where vPA = −rwiˆ

and vA = viˆ .

1-25
Problems and Solutions in Rotational Mechanics

This yields
v v
r= = = 2l .
w ( )v
2L

In this case the IC remains outside the given rod but we can imagine it to
be inside the extended rod.

(ii) vP and vQ are given:


When velocities of any two points are given, let us find the IC. We have
three following cases in this category.
Case 1. vP and vQ are arbitrary:
Let the point O be the IC taken inside the body for the sake of simplicity.
As O is instantaneously stationary, both the given points P and Q must
move perpendicular to OP and OQ, respectively. So, the angular velocity of
the body can be given as

w = (v P / OP) = vQ / OQ.

This tells us that if we drop the perpendiculars from P and Q with their line
of motion (or velocities vP and vQ) their point of intersection O would be
our instantaneous axis or center.

1-26
Problems and Solutions in Rotational Mechanics

Example 15 A rod AB of length l is leaning against the wall making an angle q with
the wall. If the bottom end of the rod moves with a constant velocity of v towards the
right, find (a) the location of the IC, (b) the angular velocity of the rod, (c) the
acceleration of the IC, and (d) the locus of the path followed by the IC.

Solution

(a) When we drop the perpendiculars to u and v at the ends A and B of


the rod, they intersect at the point C. Then C is the IC. The
coordinates of C are given as x = l sin q and y = l cos q .
(b) Since the IC of the rod is C and it is at rest instantaneously, the
u v v
angular velocity of the rotating rod is w = AC (= BC ) = l cos q
anti-
clockwise.
(c) The acceleration of C is given as
  
a C = a CB + a B.

1-27
Problems and Solutions in Rotational Mechanics


Since the point B moves with a constant velocity, putting aB = 0, we
have
a C = a CB = (BC)w 2
v2 v2
= (BC)(v / BC)2 = = .
BC l cos q

The direction of a C is vertically downwards.
(d) Taking the sum of the squares of the coordinates, the locus of C is
given as
x 2 + y 2 = (l sin q )2 + (l cos q )2 = l 2.
This is an equation of a circle PCQ having its center at the origin O.

Case 2. vP and vQ are parallel:


Let us assume that two points P and Q of the plate move in same direction
with velocities vP and vQ, respectively. Following the last procedure if we
drop the perpendiculars from the line of motion of the given points P and Q,
they meet at infinity. Does this mean that the IC is located at infinity?
Obviously not. So we cannot adopt the previous method in which we
considered the point of intersection of the perpendiculars as the IC.

1-28
Problems and Solutions in Rotational Mechanics

Then we need to choose the IC point O taken inside or outside the body.
As O is instantaneously stationary, both the given points P and Q must
move perpendicular to OP and OQ, respectively. So the angular velocity of
the body can be given as
w = v P / OP = vQ / OQ,
or
OP / OQ = v P / vQ .
This tells us that the dotted line joining the tips of the velocities will intersect
the extended line PQ at O, which is the IC. This means the location of the IC
can also be calculated geometrically using two similar triangles, as shown in
the figure.
Case 3. vP and vQ are anti-parallel:
In this case also, if we drop the perpendiculars from the velocities at P
and Q, they meet at infinity. So we have to adopt the last procedure of
connecting the tips of the velocity vectors by a straight line (dotted) that
intersects the line PQ at the point O which is our IC. The angular velocity of
the body can be given as
w = v P / OP = vQ / OQ,
or
OP / OQ = v P / vQ .
So also in this case we can find the location of the IC by using two similar
triangles, as shown in the figure.

Example 16 A cylinder of radius R rolls without sliding on a plank P which moves


with a velocity 3vo on a horizontal ground. If the center O of the cylinder moves with

1-29
Problems and Solutions in Rotational Mechanics

a velocity vo as shown in the figure, find (a) the position of the IAR from the center
of the cylinder and (b) the angular velocity of the cylinder.

Solution

(a) As the body rolls on the plank P, the lowest point Q of the cylinder
will move with a velocity equal to the velocity of the plank. Thus the
velocities of the points P and O of the cylinder are given as vQ = 3vo
and vO = vo in the +x-direction.

Connecting the tips of the velocity vectors by a straight line (dotted) which intersects
the line OQ at the point C which gives the IC. Then the angular velocity of the body
can be given as
w = vO / OC = vQ / QC,
or
QC vQ 3v
= = o = 3,
OC vO vo
or
QO + OC
= 3, where QO = R .
OC

1-30
Problems and Solutions in Rotational Mechanics

This tells us that the IC is located at a height OC = R/2 as shown in


the figure.
(b) Putting vO = vo and OC = R/2 in the expression w = (vO /OC), we
have w = 2vO /R in an anti-clockwise sense.

Example 17 The IC of a rolling body on a fixed surface:


If we photograph a running bicycle wheel from the ground frame using an
ultrafast hi-tech camera, how does bicycle wheel appear?
Solution
The condition of rolling of a body on a fixed surface is that the lowest point of the
body must be instantaneously stationary. So any point of the body must move
perpendicular to the line joining the point to the lowest point of the body, as shown
in the figure. If the distance of any point Q, say, of the body from its lowest point P is
r, the velocity of Q is given as v = r w. Since the angular velocity w is the same for all
points relative to the lowest point P, the speed of the point Q is directly proportional
to its distance r from A. In other words, the farther points from P move faster than
the nearer points. In other words, the points farther from P look more blurred
compared to the points nearer to the lowest point P, as shown in the figure. Hence,
we can see the lowest point P as a clear dot in the photograph.

1.14 Rotation about a point and gyroscopic motion


There are many instances such as a spinning top, pedestal, or table fan in which the
body rotates about a rotating axis. In these types of motion, one point of the body is
fixed. For the spinning top the lowest point does not move. This sort of motion is
called unconstrained motion about a point. We will discuss this motion as a
gyroscope in the dynamics of a rigid body.

1-31
Problems and Solutions in Rotational Mechanics

Example 18 The bob of a conical pendulum with a bob of mass m has an angular
velocity w1 and w2 relative to the center O of the circle and the point of suspension P,
respectively. Find the value of w1/w2 .

Solution
During the time dt let the bob swing at an angle d∅ and db relative to O and P,
respectively. This means the string rotates at an angle d∅ about the point P and an
angle db about the y-axis. So the angular velocity of the string is equal to the angular
velocity of the bob relative to the point of suspension P. Then we can write
db d∅
= w1 and = w 2,
dt dt
or
w1 db
= . (1.8)
w2 d∅
Let the bob undergo a small displacement d s during a time dt , which can be given as
ds = ld∅ = Rdb ,

1-32
Problems and Solutions in Rotational Mechanics

where R is the radius of the circle described by the bob. Then we have
db l
= = cosecq . (1.9)
d∅ R

Using equations (1.8) and (1.9) we have


w1
= cosecq .
w2
Problems
Calculation of the velocity and accelerations of rigid bodies

Problem 1 A rod AB of length l = 2 m is leaning against a vertical wall. If the end B


moves with a velocity of v = 4 m s−1 towards the right, find (a) the angular velocity
of the rod, and (b) the angular acceleration of the rod assuming a = the acceleration
of the point B = 8.5 m s−2 to the right.

1-33
Problems and Solutions in Rotational Mechanics

Solution

(a) The position of B is given by


x = l sin q
dx dq
⇒ = l cos q .
dt dt

⇒ v = wl cos q
v
⇒w= . (1.10)
l cos q
Putting the numerical values, we have
v 4
⇒w = =
l cos q 2 cos 37°
4
= = 2.5 rad s−1.
2 × 4/5

(b) Differentiating ω with respect to time, we have


dw 1 d
a= = (vsecq )
dt l dt
v dq secq ⎛ dv ⎞
= ⎛secq tan q ⎞ +
l⎝ dt ⎠ l ⎝ dt ⎠
(1.11)
v dq asecq
= secq tan q +
l dt l
secq
= (vw tan q + a) .
l

1-34
Problems and Solutions in Rotational Mechanics

Using the last equations (1.10) and (1.11),


secq ⎛ ⎛ vsecq ⎞
a= v ⎜ tan q + a⎞ ⎟

l ⎝ ⎝ l ⎠ ⎠

secq ⎛ v 2secq tan q


⇒a= ⎜ + a⎞ . ⎟

l ⎝ l ⎠
Evaluating the factors, we have

⇒a = sec37 °
2 ( (4)2 sec37 ° tan 37 °
2
+ 8.5 )
⇒ a = 10 rad s−2 .

Problem 2 In the previous problem, if the end B is moved with a constant velocity of
4 m s−1 to the right, find (a) the velocity and (b) the acceleration of A. You can use
all required data of the last problem.

Solution

(a) Referring to the last problem, we have


v
w= . (1.12)
l cos q
The position of A is given by
y = l cos q

dy dq
⇒ = − l sin q .
dt dt
⇒ v′ = − wl sin q . (1.13)
Using equations (1.12) and (1.13)
v′ = − (v / l cos q )l sin q = − v tan q . (1.14)

1-35
Problems and Solutions in Rotational Mechanics

Putting in the numerical values, we have


v′ = − v tan q = − (4)tan 37°

=− (4)(3 / 4) = − 3 m s−1.
The negative sign signifies that the end A moves down.
(b) Differentiating v′ with respect to time, we have
d v′ d
aA = = (v tan q )
dt dt
dq
= v⎛sec 2q ⎞ = vwsec 2q
⎝ dt ⎠
v sec q ⎞sec 2q = v sec q .
2 3
= v⎛
⎝ l ⎠ l

Evaluating, we have

(4)2 (5 / 4)3
aA = a′ = (125 / 8) m s−2 .
2
N.B. we obtained the velocity of A as
v′ = − v tan q .
As the end B moves towards the right, the angle θ increases; so tanθ will
increase. This means that the point A speeds up in the downward direction
and hence its acceleration will point vertically down.

Problem 3 Referring to the last diagram, let us assume that the rod is massless and
the particles A and B have masses m and 2m, respectively. Find the velocity of the
center of mass of the rod–particles system for θ = 45°. Put v = 3 m s−1.

1-36
Problems and Solutions in Rotational Mechanics

Solution
In the last problem, we have obtained the velocities of the points A and B as
follows:
v′ = − v tan q( ↓ )

v B = + v(→) .
Then the velocity of the center of mass of the system is

→ m viˆ + mA v′( − jˆ )
vC = B
mA + m B

2mviˆ + mv′( − ˆj )
⇒→
vC =
2m + m

v′
⇒→
2
vC = viˆ − ˆj . (1.15)
3 3
Alternatively, by equating the velocities at the ends of the rod along its length, we
have
v′ cos q = v sin q

v sin q
⇒ v¢ = = v tan q . (1.16)
cos q
Using equations (1.15) and (1.16) we have
→ v
vC = [2iˆ − tan qˆj ] , where
3
v = 3 m s−1 and tan q = 1; we have

uC = (2iˆ − ˆj ) m s−1.

1-37
Problems and Solutions in Rotational Mechanics


Problem 4 A bar AB of length l has angular velocity w = wk̂ and angular

acceleration a = ak . Find (a) the velocity, and (b) the acceleration of the mid-
point of the rod.

Solution

(a) Let the coordinates of the mid-point of the rod be x and y, respectively,
given as
x = (l / 2)sin q (1.17)

y = (l / 2)cos q . (1.18)
Differentiating both sides with time, we have
wl
vx = cos q . (1.19)
2

1-38
Problems and Solutions in Rotational Mechanics

wl
vy = sin q . (1.20)
2
Then the center of mass has the velocity

v =→vx + →vy
wl wl
= cos qiˆ − sin qjˆ (1.21)
2 2
wl
= (cos qiˆ − sin qjˆ ) .
2

(b) Differentiating the velocities in equations (1.19) and (1.20) with time, we
have
1
ax = (a cos q − w 2 sin q ) (1.22)
2

l
ay = + (w 2 cos q + a sin q ) . (1.23)
2
The acceleration of the center of mass is

a =→ a +→ a = a iˆ − a jˆ
C x y x y
l l
= (a cos q − w 2 sin q )iˆ − (w 2 cos q + a sin q )jˆ
2 2
l
= [(a cos q − w 2 sin q )iˆ − (a sin q + w 2 cos q )jˆ ] .
2

Problem 5 A slender bar AB of length l = 0.5 m is moving in a vertical plane as


its lowest point B is pulled with a constant (a) velocity v = 0.5 m s−1 and

1-39
Problems and Solutions in Rotational Mechanics

(b) acceleration a = 1 m s−2 towards the right. Find the angular acceleration of the
rod as a function of time. Assume that the rod was initially in a near vertical position
and after a time t it makes an angle θ with the horizontal.

Solution
(a) For the end B moving with v = constant ,
x = vt

⇒ l cos q = vt
dq
⇒ − l sin q =v
dt
dq v
⇒ = − cosec q .
dt l
Then
d2q v d
2
=− (cosec q )
dt l dt
v dq
= − ( − cosec q cot q )
l dt
v v
= + cosec q cot q ⎛ − cosec q ⎞
l ⎝ l ⎠
v2
= − 2 cosec 2q cot q .
l
2
Since q decreases a = − d 2q
dt

v2
⇒a= cosec 2q cot q
l2

1-40
Problems and Solutions in Rotational Mechanics

v 2 cos q
= , where
l 2 sin3q

vt v 2t 2
cos q = and sin q = 1−
l l2

v2 vt
⇒a= 3
l2 (l 2 − v 2t 2) 2

l
l3

v3t
⇒→
a = 3

(l 2
−vt 2 2
) 2

(1 / 2)3t
⇒→ kˆ ⎛ v = 1 / 2 and l = ⎞ ⇒ →
1 t
a = 3
a = 3
kˆ .
{(1/2) 2
− (1 / 2) t
2 2
}2 ⎝ 2⎠ (1 − t 2 ) 2

(b) For constant acceleration, x = 12 at 2 ,


1 2
⇒ l cos q = at
2
dq
⇒ − l sin q = at
dt
dq at
⇒ = − cosec q .
dt l

1-41
Problems and Solutions in Rotational Mechanics

at 2 4l 2 − a 2t 4
Since cos q = 2l
, sin q = 2l
,
2l
then cosec q =
4l 2 − a 2t 4

dq at 2l
⇒ =−
dt l 4l 2 − a 2t 4
2at
=−
4l 2 − a 2t 4
2t 1
= = ⎛ a = 1 and l = ⎞
1−t 4 ⎝ 2⎠

d2q d 2t
⇒ = − ⎛⎜ ⎞

dt 2 dt ⎝ 1 − t 4 ⎠
−4 t 3
1 − t 4 − t / 2⎛ ⎞
⎝ 1 − t4 ⎠
=− 2
1 − t4
1 − t 4 + 4t 4
=− 2 3
(1 − t 4 ) 2
1 + t 4)
=− 2 ( 3
.
(1 − t 4 ) 2
2
Since a = − d 2q (because θ decreases with time t) we have
dt

2 1 + t 4)
a= ( 3
.
(1 − t 4 ) 2

1-42
Problems and Solutions in Rotational Mechanics

Problem 6 A rod of length l is inclined at an angle q , as shown in the figure. Find the
locus of (a) the mid-point and (b) the point at a distance b from the top end.

Solution
(a) If the point C is the mid-point of the rod, let the coordinates of C be x, y,
l
then x = cos q
2
l
and y = sin q .
2
2 2
2x 2y
Then cos2q + sin2q = ⎛ ⎞ + ⎛ ⎞
⎝ l ⎠ ⎝ l ⎠
l2
⇒ x2 + y2 = .
4
The locus is a circle.
(b) If P is any other point except the mid-point,
then AP = mAB and BP = n AB,
where m and n are the fractions (m + n = 1).
The coordinates are given as

x = APcosq = mABcosq = ml cos q


y = BPsinq = n ABsinq = nl sin q .
x 2 y 2
Then ( ) +( )
m n
= l 2(cos2 q + sin2 q )

x2 y2
⇒ 2
+ 2 = l 2.
m n

1-43
Problems and Solutions in Rotational Mechanics

b l−b
It is an ellipse, where m = l
and n = l

x2 y2
⇒ + = l2
b 2 b 2
() l (1 − ) l

x2 y2
⇒ + = 1.
b2 (l − b)2

Problem 7 The bottom of the rod AB of length l is pulled with a constant velocity v
while its other end slides over the edge of a fixed cubical wedge W of height h. Find
(a) the angular velocity of the rod as the function of q and (b) the angular velocity of
the rod when it leaves the edge of the wedge by assuming h = l .
3

Solution
h
(a) Let AC = x. Since x
= tan q , we have

x = h cos q
dx d
⇒ = h (cot q )
dt dt
dq
⇒ v = − h cot q cosec 2q
dt
dq v vsin3q
⇒ =− = − (negative sign signifies a clockwise rotation). (1.24)
dt h cot q cosec 2q h cos q

(b) When the rod remains tangential to the wedge for the last time,
h 1
tan q = = (h = l / 3 )
l 3
⇒ q = 30°, then we have
1 3
sin q = and cos q = .
2 2

1-44
Problems and Solutions in Rotational Mechanics

Putting these values in equation (1.24), we have


dq vsin330°
⇒ =
dt h cos 30°
1 3

=
v ()
2
=
v
×
2
h ( )
2
3 8h 3


w =
v
( − kˆ ) .
4 3R

Problem 8 The bottom of the rod of length l is pulled with a constant velocity v as its
other end slides over a semi-cylindrical wedge of radius R. The wedge is moved with
a velocity of 2v. Find the angular (a) velocity and (b) acceleration of the rod. q = 37°.

Solution
R
(a) Let AC = x. Since x
= sin q , we have

x = R cosec q
dx d
⇒ = R (cosecq )
dt dt
dq
⇒ vrel = −R cosecq cot q
dt
dq vrel v sin2q
⇒ =− = − rel .
dt R cot q cosec q R cos q

1-45
Problems and Solutions in Rotational Mechanics

Putting the vrel = vA + vC = v + 2v = 3v and θ = 37°, we have


dq 3vsin2 (37°)
⇒ =− = ( − 27v / 20)R (negative sign signifies a clockwise rotation) .
dt R cos(37°)

(b) The angular acceleration of the rod is


d2q d 3v sin q tan q ⎞
= ⇒ =− ⎛
dt 2 dt ⎝ R ⎠
= (3v / R){(3 / 5)(5 / 4)2 + (3 / 4)(4 / 5)}( − 29v / 20R)
= (9963 / 1600)(v / R)2 ;
positive sign signifies an anticlockwise angular acceleration.

Problem 9 A point source P of light moves horizontally at a height H from ground


level. A step MOQ of height h casts its shadow MQ in the light emanating from the
point source. (a) Find the angular velocity of the ray POQ that sweeps the edge O of
the step, as a function of q .
(i) if the point Q moves with a constant velocity vQ = u ;
(ii) if the point P moves with a constant velocity v; (b) find the velocity of the
point Q if the point moves with a velocity v at the given instant.

1-46
Problems and Solutions in Rotational Mechanics

Solution
(a)
x
(i) Let MQ = x. Since h
= tan q , we have

x = h tan q

dx d
⇒ = h (tan q )
dt dt
dq
⇒ vQ = hsec 2q
dt

dq vQcos2q
⇒ = . (1.25)
dt h

y
(ii) Let ON = y. Since H−h
= tan q we have
y = (H − h)tan q
dy d
⇒ = (H − h) (tan q )
dt dt
dq
⇒ v P = v = (H − h)sec 2q
dt

dq vcos2q
⇒ = . (1.26)
dt H−h

(b) Using equations (1.25) and (1.26) we have


u v
⇒ = . (1.27)
h H−h

1-47
Problems and Solutions in Rotational Mechanics

Equation (1.27) can be obtained by using the properties of a triangle.


Between two similar triangles PQM and OQN, we have
ON ⎛ y ⎞ H−h
⇒ ⎜= ⎟ =
MQ ⎝ x ⎠ h

dy / dt H−h
⇒ =
dx / dt h

v H−h
⇒ =
u h
h
⇒u= v.
H−h
Problem 10 In a slider crank arrangement, a small ball P is constrained to move
along a rotating tube OP and a fixed circle of radius R. The tube is pivoted smoothly
at O. The angle b made by OP with the horizontal (x-axis) changes with time as
b = kt 3. Find the acceleration of the ball (a) as the function of time and (b) at t = 1 s.
Put the radius of the circle as R = 0.5 m and k = 1/3.

Solution
(a) It is given that
b = Kt 3.

1-48
Problems and Solutions in Rotational Mechanics

By geometry q = 2b and b = kt 3 (given), so we have


q = 2kt 3
dq
= w1 = 6kt 2 (1.28)
dt
d2q
⇒ = 12kt
dt 2
⇒ a1 = 12kt . (1.29)
Then the total acceleration of the ball is the sum of tangential and radial
(centripetal) acceleration, given as
⇒a= (Ra1)2 + (Rw1 2)2

⇒ a = R a1 2 + w1 4 . (1.30)

From equations (1.28), (1.29), and (1.30), we have


⇒ a = R (12kt )2 + (6kt 2)4

⇒ a = 12R kt 1 + 9k 2t 6 . (1.31)
Putting R = 1/2, k = 1/3, and t = 1, we have
⇒ a = 12(1 / 2)(1 / 3)(1) 1 + 9(1 / 3)2 (1)6

⇒ a = 2 2 m s−2 .
Problem 11 A laser torch is located at the origin O at a distance d from the vertical
wall. At time t = 0, the laser beam falls perpendicular to a vertical wall situated at a
distance d from the origin. (a) If the torch rotates at a constant clockwise angular
velocity ω, find (i) the velocity and (ii) the acceleration of the laser point P on the
vertical wall. (b) If the velocity of the laser point P on the vertical wall moves with a
constant upward velocity v, find the angular velocity of the torch.

1-49
Problems and Solutions in Rotational Mechanics

Solution

(a)
(i) If w = constant , q = wt ,
then y = d tan q ,
where
q = wt
⇒ y = d tan wt

dy
⇒ vy = = wd sec 2wt .
dt

(ii) Differentiating velocity with time, we have


duy d
ay = = wd (sec 2wt )
dt dt

{ d
= w 2secwt , (secwt )
dt }
= 2dw 2sec 2wt tan wt .

(b)
(i) If v is a constant, PM = y = vt
y vt
⇒ tan q = =
d d
d v dt
⇒ (tan q ) =
dt d dt

1-50
Problems and Solutions in Rotational Mechanics

dq v
⇒ sec 2 =
dt d
u cos2 q
⇒w= .
d
(ii) The angular acceleration is
w dw 1
a= = d (w 2 )
dq 2
u2 d
= 2 (cos4q )
2d d q
2u 2
= − 2 sin q cos3q .
d

Problem 12 Referring to a crank slider mechanism, P is constrained to move along


the straight inclined groove of the cranks OP and QP with angular velocities ω1 and
ω2, as shown in the figure. Write the relevant kinematical equations of P.

Solution
Let the components of the velocity of P along the x- and y-axes be vx and vy,
respectively.

1-51
Problems and Solutions in Rotational Mechanics

Resolving vx and vy along (parallel to) OP and QP, we have


dl1
= vx cos q1 + vy sin q1 (1.32)
dt

dl2
= vx cos q2 + vy sin q2. (1.33)
dt
Resolving vx and vy perpendicular (transverse) to OP and QP, we have
dq1
l1 = l1w1 = vx sin q1 − vy cos q1 (1.34)
dt

dq 2
l2 = l2 w 2 = vx sin q2 − vy cos q2. (1.35)
dt
dl dl
We have six unknown quantities given as w1, w2, vx , vy , dt1 , and dt2 .
If any two unknown quantities are given, we can solve for the other four unknown
quantities using the last four equations.

Pure rotation: a pulley–belt system

Problem 13 Three discs of radii a, b, and c are pivoted at fixed points so that they
can rotate with angular velocities of same direction. The vector sum of their angular

velocities is given as →
w = w k . Find (a) the speed of any point of the belt connecting
these three discs and (b) angular velocities of the discs.

Solution

(a) Let the speed of any point on the string be v. Since the belt does not slip, the
angular velocity of each will be directly proportional to the reciprocal of its radius.
So we can write the angular velocities of A, B, and C, respectively, as follows:
v v v
w1 = , w 2 = , w 3 = .
a b c

1-52
Problems and Solutions in Rotational Mechanics

Since w = w1 + w2 + w3
1 1 1
w = v⎛ + + ⎞
⎝a b c⎠

abc w
⇒v= .
ab + bc + ac

(b) Then the angular velocity of A is


v bc w
w1 = = .
a ab + bc + ac
Likewise, the angular velocities of B and C are given as follows:
v acw
w2 = =
b ab + bc + ac
u abw
w3 = = .
c ab + bc + ac

Problem 14 Two spinning discs A and B of radii a and b are connected by a belt. If
the belt does not slip on the discs and the maximum magnitude of the relative
acceleration between two points on the perimeters of the discs is ao, find (a) speed of
the belt, (b) the angular velocities w1 and w2 of the discs.

Solution

(a) Let the speed of any point on the string be v. Since the belt does not slip, the
angular velocity of each will be directly proportional to the reciprocal of its
radius. So we can write the angular velocities of A and B, respectively, as
follows:
v v
w1 = and w 2 = .
a b
The relative acceleration between any two points of the discs will be
maximum when the perpendicular distance between the points is minimum

1-53
Problems and Solutions in Rotational Mechanics

and maximum. Hence the chosen points can be P and Q. The relative
acceleration between P and Q is the sum of their centripetal accelerations
because they point opposite to each other:
a rel = a P + a Q = u 2/ a + v 2 / b.
Putting u = v as the belt does not slip with the discs, we have
a rel = v 2 / a + v 2 / b = v 2(1 / a + 1 / b) .
Putting arel = ao, we have
⇒ v 2(1 / a + 1 / b) = ao

⇒v= ao ab / (a + b) .

(b) Then the angular velocity of A is


w1 = v / a = ao b / a(a + b) .
The angular velocity of B is
w2 = v/b = ao a / b(a + b) .

Problem 15 A rod of length l is pivoted at the point O as shown in the figure. The
angle q made by the rod with the horizontal changes with time as q = qo sin bt . Find
(a) the acceleration of a point P of the rod as a function of time, (b) the acceleration
of point P at the mean and extreme positions, and (c) the maximum angular
frequency of revolution of the point P. Assume that at time t = 0, q = 0.

1-54
Problems and Solutions in Rotational Mechanics

Solution

(a) The point P oscillates obeying the relation


q = qo cos bt .
The angular velocity of P is
dq
w rev = = − bqo sin bt .
dt
The angular acceleration is
dw rev
a= = − b 2qo cos bt .
dt
The point P moves in a circle of radius l with variable angular velocity and
angular acceleration.
Then the radial acceleration of P is
2
ar = lw 2rev = − l{bqo sin bt} .
Then the tangential acceleration of P is
at = la = − lb 2qo cos bt .
Then the net acceleration of P at any point of its circular path is given as

atotal = at2 + a r2
= {lb2qo cos bt}2 + {l (bqo sin bt )2 }
2

= lb 2qo cos2bt + l 2b 2q o2sin4bt .

(b) At the mean position, putting cosbt = 1, we have


atotal = lb 2qo.
At the extreme position, putting cosbt = 0, we have
atotal = lb3q o2.
(c) The maximum angular frequency of revolution is
{w rev }max = − bqo sin bt
{ max
= bqo.

1-55
Problems and Solutions in Rotational Mechanics

Problem 16 A disc of radius R is rotating about an axis passing through its center of
mass perpendicular to its plane. The axis is fixed and the disc rotates with a constant
angular acceleration α. Find the acceleration of a point of the perimeter of the disc
as the function of time t.
Solution
If the disc rotates with a constant acceleration α, its angular distance is
q = at 2 .
The angular velocity is
dq
= at .
dt
The angular acceleration is
d2q
= a (given) .
dt 2
Then the tangential and radial acceleration of the point P can be respectively given
as a t = Ra and a r = Rw 2. Then the total acceleration of P is →
aP = →
at + →ar.

⇒ aP = at2 + a r2 ; where at = Ra and a r = Rw 2

= R(at )2 = Ra 2t 2, then we have

a P = (Ra)2 + (Ra 2t 2)2


2 2
at ⎞
= Ra 1 + ⎛ .
⎜ ⎟

⎝ R ⎠

Problem 17 Three fixed pulleys A, B, and C are connected by two belts as shown in
the figure. If the belts do not slip on the pulleys and the angular velocity of the pulley
A is w , find the angular velocities of the pulleys B and C and the speed of the
connecting belts.

Solution
The angular velocity of the disc A is ω (given). Since it is connected with the pulley
B by the belt whose speed at any point is
v1 = aw ,

1-56
Problems and Solutions in Rotational Mechanics

the angular velocity of the disc B is


w1 = v1/ b. (1.36)
The stepped pulley B is connected with the pulley C by another belt whose speed is
equal to v2, given as
v2 = dw1. (1.37)
Then the angular velocity of the pulley C is
w 2 = v2 / c. (1.38)
Using the last three equations, we have
w1 = aw / b
v2 = adw / b

w 2 = adw / bc.

Example 18 A disc spinning about a fixed axis with an initial angular speed wo
experiences an angular acceleration given as a = −k w , where k is a positive
constant. Find the total number of rotations of the disc until its final speed is equal
to the initial speed.
Solution
Put the given a = −k w in the equation a = w ddwq to have

dw
w = −k w .
dq
After separating the variables, we have
w dw = −k dq .
At q = 0, w = wo, and at q = ∅, w = 0, as the disc will stop momentarily before
reversing its sense of rotation.
Then, integrating both sides,
0 ∅

∫w o
w dw = − k ∫0 dq .

Evaluating the integral and simplifying the factors, we have


∅ = 2(wo ) 2 / 3k .
3

As the disc reverses its angular velocity from zero to +wo, it must cover the same
angular distance ∅. So the total angular distance is
qtotal = 2∅ = 4(wo ) 2 / 3k .
3

Then the total number of rotations of the disc will be given as


N = qtotal/ 2p = 2(wo ) 2 / 3pk .
3

1-57
Problems and Solutions in Rotational Mechanics

Note: In this example the net angular displacement is zero but the net angular
distance is nonzero. Hence, the average speed will be nonzero even though the
average velocity will be zero.

Combined motion

Problem 19 In a slider crank system, at the given instant, angular velocities ω1 and ω2
of the rods PQ and RQ, respectively if at the given instant the velocity of the block R
which is constrained to move along the x-axis (the horizontal surface) is given as v
(pointing towards the right). Assume that the rods PQ and QR make acute angles θ1
and θ2, respectively, with the horizontal. Assume that PQ = l1 and RQ = l2.

Solution
Let the horizontal positions of Q relative to P and R relative to Q be x1 and x2,
respectively. Then the horizontal position of R relative to P is
x = x1 + x

⇒ x = l1 cos q1 + l2 cos q2.


Differentiating both sides with time,
dx dq dq
= − ⎛l1 sin q1 1 + l2 sin q2 2 ⎞
dt ⎝ d t dt ⎠

1-58
Problems and Solutions in Rotational Mechanics

⇒ v = l1w1 sin q1 + l2 w 2 sin q2. (1.39)


The vertical positions
y = l1 sin q1 + h = l2 sin q2
dq1 dq
⇒ l1 cos q1 = l2 cos q2 2
dt dt
⇒ l1w1 cos q1 = l2 w 2 cos q2. (1.40)
Using equations (1.39) and (1.40)
dx l w cos q1 ⎞
v⎛= ⎞ = l1w1 sin q1 + l2 sin q2⎛ 1 1 ⎜ ⎟

⎝ dt ⎠ ⎝ l2 cos q2 ⎠

sin q1 cos q2 + cos q1 sin q2 ⎞


⇒ v = l1w1⎛ ⎜ ⎟

⎝ cos q2 ⎠
v cos q2 v cos q1
⇒ w1 = , w2 = .
l1 sin(q1 + q2 ) l2 sin(q1 + q2 )

Problem 20 Two rods are hinged smoothly to form a composite L-shaped rod. The
upper rod is pivoted at O. The angular velocities and angular accelerations of the
rods are given. Find the velocity of Q.

Solution
(a) The velocity of Q is

vQ = →
vQP + →
vP

1-59
Problems and Solutions in Rotational Mechanics


vQ = →
vQP + →
vP

= l2 w 2( − jˆ ) + l1w1( − iˆ )

= − {l2 w 2 jˆ + l1w1iˆ} .

(b) The acceleration of P relative to O is


  
(aPO) = (aPO)t + (aPO)r = − l1a1iˆ + l1w1 jˆ
2

The acceleration of Q relative to P is


  
( ) ( ) ( )
a QP = a QP + a QP = − l2w 22iˆ − l2a2jˆ
r t

The acceleration of Q relative to O is


  
( ) ( ) ( )
a QO = a QP + aPO

Using last three equations, we have

(a ) = − l w iˆ − l a jˆ − l a iˆ + l w jˆ
QO 2
2
2 2 2 1 1
2
1 1

= − (l1a1 + l2w 22)iˆ + (l1w12 − l2a2) ˆj

Problem 21 In the following guided slider crank system the rod PQ has an angular
velocity of 3 rad s−1 as shown in the figure. Find the angular velocity of the rod QR.
Put PQ/QR = a/b = 2/3 and the angle made by QR with horizontal is 60°.

1-60
Problems and Solutions in Rotational Mechanics

Solution
The velocity of Q is aw1 and the velocity of R is v. Then, resolving the velocity
along the rod QR and equating them, we have
w1 a sin q = u cos q . (1.41)
Resolving the velocities at Q and R perpendicular to the rod QR,
v1 + v2 = bw 2

⇒ w1a cos q + u sin q = bw 2. (1.42)


From equations (1.41) and (1.42),
sin q ⎞
w1a cos q + ⎛w1a sin q = bw 2
⎝ cos q ⎠

a cos2q + sin2q )
⇒ w1 ( = bw 2
cos q
w1a
⇒ w2 = = 3(2 / 3)(2) = 4 rad s−1.
b cos q

Kinematics of rolling

Problem 22 A disc of radius R rolls without sliding on a fixed horizontal surface.


The velocity of the center O of the disc is v. Let us consider a point P at the lowest
point of the disc. At t = 0, let the point P coincide with the origin O. Find (a) the
locus equation of the point P and (b) the velocity of the point P, (c) the total distance
covered during one complete rotation of the disc, and (d) the radius of curvature of
the path traced by the point P, at the given angular position, after a time t.

1-61
Problems and Solutions in Rotational Mechanics

Solution

(a) Let us mark the lowest point P by a black marker. At time t = 0, the point P
was lying at the origin. At time t, the center C of the disc moves through a
distance vt. At the same time the point P revolves by an angle of θ relative to
C during time t. As the disc rolls without sliding, the length of the arc AP is
equal to OA. Then we can write Rθ = vt and θ = ωt.
The coordinates of P can be given as
x = vt − R sin q
y = R(1 − cos q ) .
Putting v = Rw and q = wt , we have
x = Rwt − R sin wt = R(q − sin q )

y = R(1 − cos wt ) = R(1 − cos q ) .


So the locus equation of P is a cycloid.

(b) The horizontal velocity of P is


dx dq dq
vx = = R⎛ − cos q ⎞ = Rw(1 − cos q )
dt ⎝ dt dt ⎠

⇒ vx = v(1 − cos q )

1-62
Problems and Solutions in Rotational Mechanics

dy dq
vy = = R sin q = wR sin q = v sin q ⇒ v P = vx2 + vy2 = v (1 − cos q )2 + sin2 q
dt dt

q vt
= 2v sin = 2v sin .
2 R
(c) The distance covered is
D = ∫ v P dt
T = 2pvR vt
= ∫0 2v sin dt
2R
= 8R .
(d) The radius of curvature is given as
vP2
r= .
a⊥
The velocity of P at time t is

v P = v(1 − cos q )iˆ + v sin qjˆ .
We can show that the velocity of P is perpendicular to OP.
The acceleration of P at time t is
→ v2
a P = (sin qiˆ + cos qjˆ ) .
R
The acceleration is radially inward.
Then we can write
→aP . →
v P = a P v P cos b
v3 v3
(1 − cos q )sin q + sin q cos q
R R
v2 ⎧ q⎫
= 2v sin cos b
R⎨ ⎩ 2⎬

u3 2u 3 q
⇒ sin q = sin cos b
R R 2
sin q q
⇒ cos b = q
= cos
2 sin 2 2
q
⇒b= (we can also prove it geometrically in one step referring to the last figure) .
2
v2 q
Then a⊥ = a P sin b = ⎛ ⎞sin . ⎜ ⎟

⎝R⎠ 2

1-63
Problems and Solutions in Rotational Mechanics

q 2

Hence the radius of curvature at P is r =


vP2
=
2v sin 2 ( )
a⊥ v2 q
R
sin 2
q
⇒ r = 4R sin .
2
Problem 23 A disc of radius R rolls on a horizontal surface. The center of the disc
moves with a constant acceleration → a = aiˆ . Find (a) the velocities and (b) the
accelerations of the points A, B, C, and D of the disc as the function of time.

Solution

(a) For pure rolling of a body on a fixed surface v = Rw and v = at .


The velocity of A is given as
  
vA = vAO + vO

= − Rw iˆ + viˆ

= (− Rw + v) iˆ = 0.

The velocity of B is given as


  
vB = vBO + vO

= Rw jˆ + viˆ

( )
= v jˆ + v iˆ = v iˆ + jˆ = at iˆ + jˆ ( )
The velocity of D is given as
  
vC = vCO + vO

= Rwiˆ + v iˆ = viˆ + viˆ = 2 viˆ = 2atiˆ


The velocity of D is given as
  
vD = vDO + vO = − Rw jˆ + viˆ = − v jˆ + viˆ = v(iˆ − jˆ ) = at(iˆ − jˆ )

1-64
Problems and Solutions in Rotational Mechanics

(b) For pure rolling on a fixed surface


aO = a = Ra .
For uniform accelerated motion of the center of mass,
v = at.
The acceleration of the points A, B, C and D are given as following:
Put R ω2 = v2 /R, where v= at and Ra = a .
  
aA = aAO + a O = − Raiˆ + Rw 2jˆ + aiˆ = (a − Ra)iˆ + Rw 2jˆ
= + Rw 2jˆ because a = Ra

() (
2
So, aA = vR jˆ = a 2t 2 /R jˆ )

  
( )
aB = aBO + a O = Rw 2 + a iˆ − Rajˆ =
because a = Ra
(Rw 2
)
+ a iˆ − ajˆ

v2 a 2t 2
= ⎧⎛⎜ ⎞⎟ + a⎫iˆ − ajˆ = ⎧ + a⎫iˆ − ajˆ

⎩⎝ ⎠R ⎬
⎭ ⎨
⎩ R ⎬

  
aC = aCO + a O = (Ra + a )iˆ − Rw jˆ = 2aiˆ − Rw 2jˆ
2

(because a = Ra ) = 2aiˆ − Rw 2jˆ

v2 a 2t 2 ˆ ⎞
= ⎛⎜2aiˆ − jˆ ⎞⎟ = ⎜⎛2aiˆ − j⎟
⎝ R ⎠ ⎝ R ⎠
  
( ) (
aD = aDO + a O = a − Rw 2 iˆ − Rajˆ = R − Rw 2 + a iˆ − ajˆ
because a = Ra
)

v2 a 2t 2 ⎞ ˆ
= ⎛⎜a − ⎞⎟iˆ − ajˆ = ⎛⎜a − ⎟i − aj
ˆ
⎝ R⎠ ⎝ R ⎠

1-65
Problems and Solutions in Rotational Mechanics

Problem 24 A wheel of radius R is rolling on a horizontal surface with a constant


velocity v in the rain. The water particles leave the wheel and fly as freely falling bodies
under gravity. Find the maximum possible height attained by the water particles.
Solution
Let the water particle P leave the wheel at an angular position θ as shown in the
figure. As discussed earlier, the vertical velocity of the particle P is
→vy = v sin qjˆ .

The horizontal velocity of P is



vx = (v cos q + v)iˆ.
The height attained by the particle from C is
vy2
H = R cos q + h = R cos q +
2g
v 2sin2q
= R cos q + . (1.43)
2g
dH
For M to be maximum dq
=0
2
v
⇒ 0 = − R sin q + ⎧ ⎫2 sin q cos q

⎩ 2g ⎬⎭
gR
⇒ cos q = 2 .
v
gR 2
Then, putting cos q = gR
v2
and sin q = 1− ( )
u2
in equation (1.43), we have

v 2sin2q
H= + R cos q
2g
v2 ⎧ g 2R 2
=
2g ⎨⎩
1 − ⎛⎜ 4 ⎞⎟⎫ + R
⎝ v ⎠⎬ ⎭
{ }
gR
v2
v 2 (v 4 − g 2R2) gR2
= 4
+ 2 .
2g v v
v 4 − g 2R2 gR2
Hmax = + .
2gv 2 v2

1-66
Problems and Solutions in Rotational Mechanics

v2 gR2
⇒ Hmax = + .
2g 2v 2

Problem 25 A disc of radius R is rolling on a plank while the plank moves with a
velocity −2v and acceleration a. If the point B of the disc moves with a velocity u,
find (a) the angular velocity of the disc and (b) the acceleration of B. Put R = 2r,
R = 0.5 m, a = 2.5 m s−2, and v = 0.5 m s−1 and α = 2 rad s−2.

Solution

(a) As the disc rolls on the plank, the velocity of B is


 2v + u ˆ 2v + v ˆ
w=− k=− k
R+r R + R/2
2v ˆ 2(0.5) ˆ
=− k=− k = − 2kˆ
R 0.5

(b) The vertical (radial) acceleration of B is



ay = − rw 2jˆ
The horizontal acceleration of B is

ax = {(R + r)a + a}iˆ
Then, the total acceleration of B is
  
aB = ax + ay

={(R + r)a + a}iˆ − rw 2jˆ

1-67
Problems and Solutions in Rotational Mechanics

= {(R + R / 2)a + a}iˆ − (R / 2)w 2jˆ


1
= {(3Ra + 2a)iˆ − Rw 2jˆ }
2
1
= ⎡{3(1 / 2)(2) + 2(2.5)}iˆ − (1 / 2)(2) jˆ ⎤
2
2⎣ ⎦

= 4iˆ − jˆ m s−2

Problem 26 Three uniform cylinders of radii 0.6R, R, and 1.5R are loaded on to each
other by three planks, as shown in the figure. If the planks 1, 2, and 3 move with
velocities +2v, −v, and +3v, respectively, and the cylinders roll without sliding. Find the
angular velocities of the cylinders and the velocities of the center of mass of the cylinders.

1-68
Problems and Solutions in Rotational Mechanics

Solution
As the cylinders roll without sliding, the velocities of the top and bottom of the
lowest cylinder are 2v and 0, respectively. So the angular velocity of the lowest
cylinder is
→ 2v + 0 −5 ˆ
w1 = 3
= vk .
2 × 5R 3
The velocity of A is
→ 2v + 0 ˆ
vA = i = viˆ.
2
The velocity of the top and bottom of the middle cylinder are −v and 2v,
respectively. So the angular velocity of the cylinder is
→ v + 2v ˆ 3v ˆ
w2 = k= k.
2R 2R
The velocity of B is
− v + 2v ˆ v
vB = i = iˆ.
2 2
The velocity of the top and bottom of the topmost cylinder are 3v and −v,
respectively. So the angular velocity of the cylinder is
→ 3v + v ˆ 4v ˆ
w3 = − 3
k=− k.
2× R 2
3R
The velocity of C is
3v − v ˆ
vC = i = viˆ.
2

1-69
Problems and Solutions in Rotational Mechanics

Instantaneous axis of rotation

Problem 27 A rod of length l is leaning against a horizontal surface and an inclined


plane. The rod is undergoing a combined motion in a vertical plane. When the rod
makes angles θ and β with the planes, as shown in the figure, the lowest point of the
rod moves towards the right with a velocity v. Find the angular velocity of the rod.

Solution
Method 1
If the end A of the rod AB moves with a velocity v, let the other end B of the rod
move with a velocity v¢ . Resolving the velocities along the rod and equating them,
we have
v cos q = v¢ cos b

⇒ v¢ = v cos q / cos b . (1.44)


Since the components of v and v¢ perpendicular to the rod are oppositely directed,
the relative velocity between the end A and B is
vrel = v sin q + v¢ sin b .

Then the angular velocity of the rod is


w = vrel / l = (v sin q + v¢ sin b ) / l . (1.45)

1-70
Problems and Solutions in Rotational Mechanics

Using equations (1.44) and (1.45), we have


w = {v sin q + (v cos q / cos b )sin b} / l
= v(sin q cos b + cos q sin b ) / l cos b
v sin(q + b )
= .
l cos b
Method 2
We drop two perpendiculars to the velocities v and v¢ at A and B, respectively.
They intersect at I, which is known as the IAR. In the triangle ABI using the triangle
property (sine rule), we have
⇒ AI / sin f = AB/ sin a
⇒ AI = ABsinf/ sin a .
Putting AB = l, α = θ + β, and φ = 90° − β, we have
AI = l sin(90° − b ) / sin(q + b )
⇒ AI = l cos b / sin(q + b ) .
Since the velocity of instantaneous axis of rotation I is zero, the angular velocity of
the rod can be written as
w = v / AI = v / {l cos b ) / sin(q + b )}
v sin(q + b )
w= .
l cos b

Problem 28 A sphere is spinning about the horizontal axis OQ with an angular



velocity w1. The axis OQ rotates about the vertical axis AB with an angular

velocityw1. Find the (a) angular velocity of a point P, say, on the vertical diameter
of the sphere about the point O (b) angular acceleration of the sphere.
Solution

(a) The angular velocity wP of a point P on the vertical diameter of the sphere

about the point O is equal to the angular velocity w1 of a point on the
vertical diameter of the sphere about the axis OQ plus the angular velocity

w2 of OQ about the vertical axis AB. So, we can write
  
wP = w1 + w2
  
Since w1 and w2 are mutually perpendicular, the magnitude of wP can be
given as

wP = w12 + w 22
 
The angle of orientation of wP with upward vertical or w2 is given as
w
ø = tan−1 ⎛⎜ 1 ⎞⎟
⎝ w2 ⎠

1-71
Problems and Solutions in Rotational Mechanics

 
(b) Let the sphere change its spin (angular velocity from w1(t ) to from w1(t + dt )
during a time dt. As the sphere rotates about the axis OQ, its angular

velocity is radially outward. The tip of the angular velocity vector w1 rotates

with an angular velocity w2 which is equal to the angular velocity of the axis
OQ about the fixed vertical axis AB. We can see from the vector diagram
 
that the change in angular velocity w1, that is, dw1 is tangential because the

angular velocity w1 is directed radially outward.
So, the angular accelertaion

 dw1 dw1
a = = ∅
ˆ
dt dt
tangential to the circular dotted path traced by the sphere as shown in the
figure. Then by putting
dw1 = w1dθ
we have the angular acceleration

 dw1 dw1 w dθ
a= = ∅
ˆ = 1 ∅ ˆ,
dt dt dt
where ddtθ = w2 . Then, we have the final expression

a = w1w2∅ˆ

This signifies that the angular acceleration due to the change in the
direction of angular velocity points tangentially to the path (dotted blue

circle) traced by the tip of the spin angular velocity vector w1.

1-72

You might also like